Сайт Романа ПарпалакаЗаметкиНаучный калейдоскопЧто такое суперсимметрия?

Что такое суперсимметрия?

21 августа 2010 года

Введение

LHC (большой адронный коллайдер) уже начал свою работу. Среди нескольких теорий, которые будут проверяться на LHC, не последнее место занимают суперсимметричные модели. Суперсимметрия как раз является областью моей научной деятельности, и я решил в научно-популярной форме попытаться рассказать, что же это такое.

Я расскажу в этой статье о современных теориях в физике элементарных частиц, о некоторых идеях и следствиях этих теорий. В отдельных местах изложение построено в предположении о том, что читатель не забыл школьную математику и физику. Среди рассмотренных тем следующие:

Теоретические основы физики элементарных частиц

Физика элементарных частиц — одна из немногих областей человеческого знания, где удалось проникнуть глубже всего в тайны материи и объяснить ее свойства. До сих пор сокращение числа законов, описывающих мир, было одной из основных тенденций при построении научных теорий. При этом главной целью всегда оставалось и остается построение единой теории поля, которая бы объединила все знания человечества о природе, и из которой можно было бы вывести (хотя бы в принципе) все законы как частные случаи такой теории.

Фундаментальные взаимодействия

В настоящее время известно четыре фундаментальных взаимодействия: гравитационное, электромагнитное, сильное и слабое. Первые два обладают дальнодействием и проявляются в повседневной жизни. Гравитация, например, управляет движением небесных тел. Все мы испытываем гравитационное притяжение Земли. Электромагнетизм объясняет большинство явлений, с которыми сталкивается человек в повседневной жизни. Два других взаимодействия короткодействующие. Они проявляются только на масштабах атомного ядра (объясняют альфа- и бета-распад) и становятся определяющими на более мелких масштабах.

В микромире ключевую роль играют квантовые свойства частиц. Для описания фундаментальных взаимодействий, однако, недостаточно обычной квантовой механики. Во-первых, квантовая механика является нерелятивистской теорией, то есть она верна для малых скоростей по сравнению со скоростью света. Во-вторых, квантовая механика не описывает процессы рождения и уничтожения частиц, которые происходят при взаимодействии частиц высоких энергий. Релятивистским обобщением (согласующимся с идеями специальной теории относительности) квантовой механики является квантовая теория поля.

Квантовая теория поля

В квантовополевых теориях частицы материи являются «квантами» (возмущениями) соответствующих полей. Взаимодействие между частицами переносится специальными полями. Предполагается, что частицы материи в процессе взаимодействия испускают и поглощают другие частицы — кванты поля-переносчика.

Первый успешный пример квантовой теории поля — квантовая электродинамика — был построен в работах Фейнмана, Швингера и Томонаги в середине двадцатого века, за что они были удостоены Нобелевской премии в 1965 году. Квантовая электродинамика рассматривает взаимодействие между заряженными частицами (например, электронами и позитронами), возникающее вследствие обмена фотонами — квантами электромагнитного поля.

Вплоть до настоящего времени квантовая электродинамика остается самой точной физической теорией. Теоретические расчеты в рамках квантовой электродинамики совпадают с результатами экспериментов с точностью до 10−10.

Симметрия в физике элементарных частиц

Под симметрией физики понимают неизменность чего-либо при выполнении определенных преобразований. При этом большую роль играет симметрия законов, или уравнений. Например, уравнение x2 = 4 симметрично (то есть не меняется) относительно преобразования x → − x.

В физике симметрии играют двоякую роль. Во-первых, каждому типу симметрии физической системы соответствует сохраняющаяся величина. Например, из однородности времени (неизменность относительно преобразований t → t + Δt) следует закон сохранения энергии, из однородности пространства (неизменность относительно преобразований координат x → x + Δx) — закон сохранения импульса, из изотропности пространства (неизменность относительно поворотов) — закон сохранения момента импульса (момент импульса L = mvr характеризует «количество вращения» и является аналогом импульса p = mv).

Во-вторых, от новых физических теорий можно требовать выполнения различных симметрий. Чем больше таких требований — тем меньше произвол в построении теории.

Примером физической теории, обладающей симметрией, является обычная квантовая механика, оперирующая волновыми функциями. Волновая функция частицы — это комплексная функция, например, пространственных координат (грубо говоря, комплексное число в каждой точке). Ее можно рассчитать из уравнения Шрёдингера. Физический смысл имеет не сама волновая функция, а квадрат ее модуля, который показывает вероятность нахождения частицы в каком-либо месте.

Если все волновые функции умножить на одно и то же комплексное число с модулем 1, никакие предсказания теории не изменятся. Действительно, модуль произведения комплексных чисел равен произведению модулей, и от такого домножения никакие вероятности не изменяются.

Это пример так называемой глобальной симметрии (глобальной — потому, что волновая функция умножалась в разных точках на одно и то же число). Суть этой симметрии заключается в том, что теория не изменяется относительно некоторого класса преобразований (в нашем случае эти преобразования — умножение на произвольное комплексное число с модулем, равным 1).

Квантовая электродинамика обладает симметрией относительно преобразований, называемых калибровочными. Эти преобразования заключаются в домножении поля электронов на комплексное число с модулем 1 (правда, чтобы теория не изменялась, одновременно с преобразованием поля электронов нужно выполнить и некоторые другие преобразования электромагнитного поля). В отличие от рассмотренного выше случая квантовой механики, это число уже может быть в каждой точке различным (локальная симметрия).

Интересно отметить следующий момент. Как было сказано выше, с каждой симметрией связана сохраняющаяся величина. В случае калибровочных преобразований квантовой электродинамики такой сохраняющейся величиной является обычный электрический заряд.

В пятидесятых годах Янг и Миллс построили модель, уравнения которой не менялись под действием более сложных локальных калибровочных преобразований. Сначала интерес был исключительно математическим. Однако потом на основе теории Янга — Миллса были созданы важнейшие теории взаимодействия элементарных частиц — теория электрослабых взаимодействий и квантовая хромодинамика. Эти теории, обладающие калибровочной симметрией, получили экспериментальное подтверждение.

Стандартная модель фундаментальных взаимодействий

В шестидесятых годах удалось объединить электромагнетизм и слабые взаимодействия. Салам, Глэшоу и Вайнберг построили теорию электрослабых взаимодействий. В 1979 году им была присуждена Нобелевская премия. Новая теория предсказала существование новых частиц, так называемых W- и Z-бозонов. Они отвечают за «перенос» слабого взаимодействия. Эти бозоны были открыты на протонном суперсинхротроне в 1983 году.

Казалось бы, каким образом можно объединить электромагнитные и слабые взаимодействия, если у первых радиус взаимодействия бесконечен (действительно, мы видим свет — электромагнитное излучение — от удаленных галактик и других астрономических объектов), а у вторых он не превышает размеры атомного ядра? Оказывается, такая «несимметричность» связана с тем, что масса фотонов равна нулю, а масса W- и Z-бозонов очень большая, они примерно в 100 раз тяжелее протона.

Нарушение так называемой электрослабой симметрии является важным свойством теории электрослабых взаимодействий (этой симметрией обладают уравнения теории). В результате нарушения W- и Z-бозоны и некоторые другие частицы (например, электроны) приобретают массы.

В рамках модели Янга — Миллса калибровочные бозоны нельзя сделать массивными, не разрушив калибровочную симметрию. Для нарушения электрослабой симметрии был придуман механизм Хиггса. Основная идея заключается в том, что все пространство пронизывает специальное хиггсовское поле, которое взаимодействует с остальными полями и нарушает симметрию, хотя уравнения теории остаются симметричными. Возмущения хиггсовского поля должны проявляться на эксперименте как новые частицы — хиггсовские бозоны. Бозон Хиггса — очень тяжелая частица, тяжелее W- и Z-бозонов. Поэтому она пока не открыта экспериментально.

Теория сильных взаимодействий, квантовая хромодинамика, тоже основана на уравнениях Янга — Миллса. Квантовая хромодинамика говорит, что многие элементарные частицы — мезоны и барионы (например, протон) — состоят из кварков. Однако изолированные кварки никогда не наблюдались (это явление называется конфайнментом). Из-за сложности уравнений квантовой хромодинамики конфайнмент до сих пор не выведен из них напрямую. Кстати, решение уравнений Янга — Миллса и объяснение конфайнмента является одной из семи проблем тысячелетия, за которые институт Клэя назначил приз в миллион долларов.

Квантовая хромодинамика также находит подтверждение в ускорительных экспериментах. Стандартная модель фундаментальных взаимодействий включает в себя модель электрослабых взаимодействий и квантовую хромодинамику. Стандартная модель оказалась в состоянии объяснить практически все экспериментальные данные, полученные к настоящему времени в физике элементарных частиц.

Суперсимметрия

Идея суперсимметрии

Перед тем, как перейти к обсуждению суперсимметрии, рассмотрим понятие спина. Спин — это собственный момент импульса, присущий каждой частице. Он измеряется в единицах постоянной Планка и бывает целым или полуцелым. Спин является исключительно квантовомеханическим свойством, его нельзя представить с классической точки зрения. Наивная попытка трактовать элементарные частицы как маленькие «шарики», а спин — как их вращение, противоречит специальной теории относительности, так как точки на поверхности шариков должны в таком случае двигаться быстрее света. Электроны обладают спином 1/2, фотоны — спином 1.

Суперсимметрия — это симметрия между частицами с целым и полуцелым спином. Идея суперсимметрии была предложена в теоретических работах Гольфанда и Лихтмана, Волкова и Акулова, а также Весса и Зумино около 40 лет назад. Вкратце она заключается в построении теорий, уравнения которых не изменялись бы при преобразовании полей с целым спином в поля с полуцелым спином и наоборот. С тех пор были написаны тысячи статей, суперсимметризации были подвергнуты все модели квантовой теории поля, был разработан новый математический аппарат, позволяющий строить суперсимметричные теории.

Стандартную модель фундаментальных взаимодействий, рассмотренную ранее, тоже можно сделать суперсимметричной. При этом решается ряд ее проблем. Рассмотрим некоторые из них.

Мотивировка суперсимметрии

Несмотря на огромные успехи стандартной модели в объяснении экспериментальных данных, она обладает рядом теоретических трудностей, которые не позволяют стандартной модели быть окончательной теорией, описывающей наш мир. Оказывается, часть этих трудностей может быть преодолена при суперсимметричном расширении стандартной модели.

Объединение констант связи

Гипотеза великого объединения, которой придерживаются многие физики, говорит, что различные фундаментальные взаимодействия есть проявления одного, более общего, взаимодействия. Это взаимодействие должно проявляться при огромных энергиях (по различным оценкам, энергия великого объединения в 1013 или даже в 1016 раз превосходит энергию, доступную современным ускорителям элементарных частиц). При понижении энергии от объединенного взаимодействия «отщепляется» сначала гравитационное взаимодействие, потом сильное, а в завершение электрослабое взаимодействие распадается на слабое и электромагнитное.

В стандартной модели, однако, электрослабое и сильное взаимодействия объединены лишь формально. Они могут оказаться разными проявлениями общего взаимодействия, а могут и не оказаться. Тем не менее, анализ экспериментальных результатов дает некоторые подсказки к вопросу о существовании великого объединения.

У каждого из фундаментальных взаимодействий есть величина, которая характеризует его интенсивность. Эта величина называется константой взаимодействия. Константа электромагнитных взаимдействий просто равна заряду электрона. В случае сильных и слабых взаимодействий ситуация несколько сложнее.

Одно из интересных свойств квантовой теории поля состоит в том, что константа взаимодействия на самом деле не константа — она меняется при изменении характерных энергий процессов с участием элементарных частиц, причем теория может предсказать характер этой зависимости.

В частности, это означает, что при приближении к электрону на расстояния, гораздо меньшие размеров атома, начинает меняться его заряд! Это изменение обусловлено квантовыми эффектами, которые подтверждены в экспериментах. Одно из первых подтверждений — наблюдение лэмбовского сдвига — небольшого смещения уровней энергии электронов в атоме водорода.

Константы электромагнитного, слабого и сильного взаимодействий измерены с достаточной точностью для того, чтобы можно было вычислить их изменение с ростом энергии. Результаты изображены на рисунке. В стандартной модели (графики слева) нет таких энергий, где произошло бы объединение констант взаимодействий. А в минимальном суперсимметричном расширении стандартной модели (графики справа) такая точка имеется. Это значит, что суперсимметрия в физике элементарных частиц обладает приятным свойством — в ее рамках возможно великое объединение!


Эволюция калибровочных констант связи с ростом масштаба энергии в стандартной модели и МССМ.

Объединение с гравитацией

Стандартная модель не включает гравитационное взаимодействие. Оно совершенно незаметно в ускорительных экспериментах из-за малых масс элементарных частиц. Однако при больших энергиях гравитация может стать существенной.

Современная теория гравитационных взаимодействий — общая теория относительности — является классической теорией. Квантовое обобщение этой теории, без сомнения, стало бы самой общей физической теорией, если бы было построено. Помимо отсутствия каких бы то ни было экспериментальных данных, имеются серьезные теоретические препятствия в построении теории квантовой гравитации.

В объединении гравитации с остальными взаимодействиями также есть трудности. Переносчик гравитационного взаимодействия, гравитон, должен иметь спин 2, в то время как спин переносчиков остальных взаимодействий (фотон, W- и Z-бозоны, глюоны) равен 1. Чтобы «перемешать» эти поля, нужно преобразование, меняющее спин. А преобразование суперсимметрии как раз и есть такое преобразование. Оно уменьшает спин частицы на 1/2 и, следовательно, может перемешивать частицы с разными спинами. Таким образом, объединение с гравитацией в рамках суперсимметрии вполне естественно.

Природа темной материи Вселенной

Суперсимметрия может объяснить некоторые результаты исследований в космологии. Один из таких результатов заключается в том, что видимая (светящаяся) материя составляет не всю материю во Вселенной. Значительное количество энергии приходится на так называемую темную материю и темную энергию.

Прямым указанием на существование темной материи являются зависимости скоростей звезд в спиральных галактиках от их расстояния до центра. Эту зависимость легко вычислить. Оказывается, экспериментальные данные существенно расходятся с предсказаниями теории.

Расхождение объясняют тем, что галактики находятся в «облаках» темной материи. Частицы темной материи взаимодействуют только гравитационно. Поэтому они группируются вокруг галактик (правильнее было бы сказать, что обычная материя группируется вокруг сгустков темной материи) и искажают распределение масс в галактике.

Реликтовое излучение — равновесное тепловое излучение, заполняющее Вселенную. Это излучение отделилось от вещества на ранних этапах расширения Вселенной, когда электроны объединились с протонами и образовали атомы водорода (рекомбинация). Тогда Вселенная была в 1000 раз моложе, чем сейчас. Нынешняя температура реликтового излучения составляет примерно 3 K.

Недавние высокоточные измерения распределения температуры реликтового излучения по небу в эксперименте WMAP показали, что общая энергия Вселенной распределена между темной энергией (73%), темной материей (23%) и обычной материей (4%), то есть темная материя составляет значительную часть, превосходящую во много раз долю видимой материи.

В стандартной модели нет подходящих частиц для объяснения темной материи. В то же время в некоторых суперсимметричных моделях есть прекрасный кандидат на роль холодной темной материи, а именно нейтралино — легчайшая суперсимметричная частица. Она стабильна, так что реликтовые нейтралино могли бы сохраниться во Вселенной со времен Большого взрыва.

Что касается темной энергии, ее природа в рамках современных физических теорий совершенно непонятна. Это настоящий вызов физикам двадцать первого века. Темную энергию можно интерпретировать как собственную энергию вакуума, однако при этом возникают огромные несоответствия между теоретическими оценками и наблюдаемым значением плотности темной энергии. Существование темной энергии приводит к наблюдаемым следствиям — ускоренному расширению Вселенной в настоящее время.

МССМ

Для построения суперсимметричных моделей был развит математический аппарат, останавливаться на котором здесь нет никакой возможности. Однако, несмотря на всю сложность математического аппарата, суперсимметричные теории обладают рядом простых особенностей.

К одной из таких особенностей относится удвоение числа частиц. Каждая частица приобретает суперпартнера — частицу, обладающую точно такими же свойствами, за исключением спина, отличающегося на 1/2.

В стандартной модели нет частиц, которые могли бы быть суперпартнерами друг друга. Следовательно, в суперсимметричных расширениях стандартной модели каждая частица приобретает своего суперпартнера — новую частицу. Минимальная суперсимметричная стандартная модель (МССМ) требует для построения меньше всего новых частиц.

Другой важной особенностью суперсимметричных моделей является нарушение суперсимметрии. Если бы такого нарушения не было, суперпартнеры имели такие же массы, что и обычные частицы. Однако новые частицы с массами известных частиц стандартной модели никогда не наблюдались. Также без нарушения суперсимметрии не работал бы хиггсовский механизм нарушения электрослабой симметрии.

Чтобы применять суперсимметричные модели в физике высоких энергий, необходимо потребовать нарушение суперсимметрии. При этом суперпартнеры могут приобрести большие массы, чем можно объяснить их ненаблюдение в настоящее время.

Конкретный механизм нарушения суперсимметрии сейчас неизвестен. Это существенно снижает предсказательную силу модели, так как в ней появляется большое число свободных параметров, подбирая которые, можно получать произвольные следствия. Некоторые соображения, например, гипотеза великого объединения, позволяют ограничить число свободных параметров. Исследование ограничений на параметры суперсимметричных моделей является одним из важных направлений в исследовании физики за пределами стандартной модели.

Экспериментальный статус суперсимметричных моделей

Суперсимметрия является одним из основных кандидатов на роль новой теории в физике элементарных частиц за рамками стандартной модели. Поиски различных проявлений суперсимметрии в природе были одной из главных задач многочисленных экспериментов на коллайдерах (LEP — большой электрон-позитронный коллайдер и Тэватрон) и в неускорительных экспериментах на протяжении нескольких десятилетий.

К сожалению, результат пока отрицательный. Нет никаких прямых указаний на существование суперсимметрии в физике элементарных частиц, хотя имеющиеся суперсимметричные модели в целом не запрещены имеющимися теоретическими и экспериментальными требованиями.

LHC (большой адронный коллайдер) — новый ускоритель, построенный в ЦЕРНе. Его энергия в семь раз превосходит энергию недавно закрытого американского ускорителя Тэватрона. В большинстве суперсимметричных моделей массы новых частиц лежат в области, доступной LHC. LHC уже открыл бозон Хиггса. Предполагается, что суперсимметричные частицы тоже будут открыты. В новых экспериментах низкоэнергетическая суперсимметрия будет либо обнаружена, либо исключена.

Хотя суперсимметрия и не открыта на опыте, различные суперсимметричные модели могут быть исследованы уже сейчас. Во-первых, следует исключить модели, в которых новые частицы имеют недостаточно большие массы, к настоящему времени уже закрытые экспериментально. Во-вторых, расхождения некоторых экспериментальных данных и теоретических предсказаний стандартной модели могут объясняться вкладом суперсимметричных частиц, и с этой точки зрения некоторые суперсимметричные модели оказываются предпочтительнее других.

Многие специалисты в физике высоких энергий исследуют различные варианты суперсимметричных моделей и их следствия. Вполне возможно, что одна из таких моделей будет подтверждена на ускорителе LHC.


Дополнение

Недавно я опубликовал эту статью на Хабрахабре, и в комментариях пользователь alexius2 задал интересные вопросы. Я привожу их здесь вместе со своими ответами.

Если не затруднит, поясните пожалуйста пару моментов:

При понижении энергии от объединенного взаимодействия «отщепляется» сначала гравитационное взаимодействие, потом сильное, а в завершение электрослабое взаимодействие распадается на слабое и электромагнитное.

Что значит «отщепляется»? Как сравниваются разные виды взаимодействий по уровню энергии?

По поводу «отщепления» взаимодействий. В большинстве наблюдаемых явлений известные взаимодействия (гравитационное, электромагнитное, слабое и сильное) независимы, проявляют себя различно. Например, у них разная интенсивность. Гравитационное притяжение двух электронов в 1039 раз меньше их электростатического отталкивания.

Суть гипотезы великого объединения в том, что эта независимость, «непохожесть», только кажущаяся, а на самом деле все эти взаимодействия — разные стороны одного взаимодействия.

Каким образом с точки зрения теории проявляется единый характер взаимодействий? Должна быть возможность «перемешать» поля, соответствующие разным взаимодействиям.

За аналогией можно обратиться к восприятию человеком расстояний. «Ширину» и «высоту» объектов мозг оценивает по размеру изображения на сетчатке глаза, а «глубину» и удаленность объектов — по разности изображений на сетчатке и напряжению в глазных мышцах. То есть «ширина» и «глубина» — это разные вещи по способу измерения. Однако по сути это одно и то же. Единый характер расстояний проявляется, если мы начнем поворачивать объект. Тогда к «ширине» добавится немного «глубины», а к «глубине» — немного «ширины», произойдет перемешивание этих величин.

С полями взаимодействий происходит то же самое. Если фундаментальные взаимодействия — разные стороны одного и того же великого взаимодействия, то должны существовать преобразования, перемешивающие поля взаимодействий, и теория должна быть симметрична относительно таких преобразований.

Такой симметрии, как я написал в начале комментария, на самом деле не наблюдается. Можно предположить, что эта симметрия должна проявляться при очень высоких энергиях (или, что то же самое, на очень малых расстояниях), а при понижении энергии в силу каких-то неизвестных механизмов симметрия разрушается и взаимодействие распадается на гравитационное, электрослабое и сильное, а затем (из-за механизма Хиггса) электрослабое распадается на слабое и электромагнитное. В этом и состоит гипотеза великого объединения.

Во-первых, следует исключить модели, в которых новые частицы имеют недостаточно большие массы, к настоящему времени уже закрытые экспериментально.

Правильно ли я понял, что открытие новых частиц с относительно небольшими массами уже принципиально невозможно (т.к. все открыты)?

В целом это так, хотя есть тонкости вроде времени жизни и интенсивности взаимодействия с остальными частицами. Долгоживущие частицы, достаточно сильно взаимодействующие с веществом, уже открыты.

Частицы с малым временем жизни в основном открывают в процессах рождения на ускорителях. Исходные частицы нужно хорошо разогнать, чтобы для рождения хватило энергии. На Хиггс, например, до сих пор энергии не хватало.

Частицы, практически не взаимодействующие с веществом, очень трудно зарегистрировать независимо от их массы.

Поделиться
Посмотрите в блоге

Читайте также

Популярно о суперсимметрии
Недавно я опубликовал на Хабрахабре свою научно-популярную статью о современной теоретической физике высоких энергий и о суперсимметрии как примере одной из теорий. Пользуясь случаем, благодарю Алика Кирилловича за приглашение на Хабрахабр и за помощь в адаптации статьи.
2010
LHC и мой диплом
Надеюсь, за полгода на LHC не успеют получить существенные результаты. А то глупо будет начинать защиту диплома со слов «Тема моей работы — минимальная суперсимметричная стандартная модель, хотя низкоэнергетическая суперсимметрия уже была исключена экспериментами на LHC.»
2009
История развития теоретической физики высоких энергий
Физика элементарных частиц (или физика высоких энергий) — это наука исключительно XX века, которая целиком основана на квантовых представлениях о мире. Ее теоретические и экспериментальные методы развивались вместе.
2011
Беседа о физике высоких энергий, бозоне Хиггса и космологии
В последнее время мне задали несколько интересных вопросов по физике, особенно в комментариях к статье «Что такое суперсимметрия».
2012

Комментарии

#1. 23 августа 2010 года, 11:58. пишет:
Спасибо за статью. Очень познавательно и интересно.
#2. 23 августа 2010 года, 20:04. пишет:
Интересно, полезно, доступно.
#3. 9 апреля 2011 года, 18:40. Василий пишет:
В ходе ознакомления со статьёй возникла мысль: что будет если в миг, в одночасье исчезнут все знания(накопленные на материальных носителях)…. Какую бездну километров придётся пройти, чтобы вновь познать некогда познанное.
#4. 12 июля 2011 года, 23:54. Вячеслав пишет:
Здравствуйте,Роман! Я не физик,просто бытовой дилетант.Хочу задать наверняка глупый вопрос. Согласно принципу неопределенности Гейзенбенрга в вакууме постоянно из ничего возникают частицы и античастицы и быстро аннигилируют.Как я понимаю,этих частиц невообразимо много. Но ведь они имеют массу и может быть это и есть темная материя?
#5. 13 июля 2011 года, 00:57. пишет:
Вячеслав, когда говорят об энергии вакуума, подразумевают эти виртуальные частицы. Но свойства энергии вакуума отличаются от свойств темной материи. В частности, темная материя собирается вместе с обычной материей в структуры вроде галактик и скоплений галактик, и в галактиках плотность темной материи гораздо больше, чем можно было бы ожидать от виртуальных частиц.

Энергия вакуума по свойствам подходит на роль темной энергии. Но здесь физики натыкаются на огромное количественное несоответствие: плотность энергии вакуума для известных частиц на 60 порядков превосходит экспериментальное значение плотности темной энергии.
#6. 13 июля 2011 года, 22:38. Вячеслав пишет:
Роман,спасибо за ответ. Теперь все более-менее понятно. У меня еще один вопрос. Сейчас общепринято,что вселенная расширяется ускоренно,однако почему-то этот вывод делается из наблюдений за разбеганием галактик глядя в прошлое этих объектов. Может быть сейчас, когда после наблюдаемого нами события прошли сотни миллионов лет, они уже замедляются?
#7. 13 июля 2011 года, 23:04. пишет:
Мы можем делать вывод о том, что происходит «сейчас», по наблюдению ближних объектов, а о том, что происходило раньше — по наблюдению удаленных. Дело в том, что в космологических моделях предполагается, что в очень больших масштабах Вселенная однородна (распределение скоплений галактик однородно), и расширение в каждой области Вселенной проходит с одним и тем же темпом. Данные наблюдений не противоречат этим предположениям.

Ускоренное расширение Вселенной впервые было обнаружено по наблюдению взрывов сверхновых определенного типа (раньше предполагалось, что Вселенная расширяется замедленно, то есть темп расширения со временем падает): http://ru.wikipedia.org/wiki/Тёмная_энергия

Затем более точные сведения были получены при обработке данных со спутника WMAP. В обработке данных предполагается, что доля темной энергии во Вселенной составляет столько-то процентов, и на основе этого предположения вычисляется, что должен был бы наблюдать этот спутник. Если доля составляет 73%, выводы теории и наблюдения наиболее близки. Вот откуда взялось это число.
#8. 14 июля 2011 года, 23:01. Вячеслав пишет:
Ну что ж,наверное темная энергия существует. Спасибо Вам за популяризацию таких сложных вещей. Ваша статья,как я понимаю,максимально изложена просто,но очень интересно и почти доступна даже для меня.Она меня подстегнула к чтению статей на эти темы. Кстати,из информации по Вашей ссылке(о горизонте событий на краю Вселенной)возникла аналогия,что мы живем в черной дыре"наоборот". Желаю Вам успехов в научной работе и,чем черт не шутит,великого открытия.
#9. 15 июля 2011 года, 19:53. пишет:
Какая-то аналогия есть. Хотя для расширяющейся Вселенной ситуация более симметричная: ни мы не сможем попасть к удаленным частицам, ни они к нам. А вот под горизонт событий черной дыры мы попасть можем.

Кстати, я вспомнил, что есть хорошая научно-популярная статья в «Науке и жизни» по проблемам современной космологии:
http://www.nkj.ru/archive/articles/687/
#10. 29 августа 2011 года, 17:21. Виталий пишет:
Здравствуйте, Роман.
У меня математика, физика на уровне технического института, поэтому хочу спросить Ваше мнение по нижеследующим вопросам.
Вот смотрите, при тензоре равном пяти теории Калуцы-Клейна удалось объединить электродинамику и гравитацию. Но что такое 5 измерение было для них непонятно. Эйнштейн в работе над единой теорией поля отказался от пятимерности, поскольку ее не «видел». В то же время на основаии пятимерной теории Калуцы-Клейна возникла М-теория.
Известно с классического примера, когда один близнец остается на Земле, а второй ускоряется от него на космическом корабле.
По мере достижения околосветовой скорости пространство-время ксмонавта по отношеию к пространству-времени Землянина растет деформация — часы космонавта замедляют свой ход, корабль вытягивается, масса растет. Но космонавт этой деформации не ощущает.
Очевидно, что пространство-время релятивистски подвижно, поэтому следует к пространству-времени: (3+1) добавить ось релятивистской его подвижности,т.е.+1:
(3+1)+1=5. Получается, общая мерность равна пяти. Форма пятой оси, как оси релятивистской подвижности пространства-времени имеет форму экспоненты.
Пожалуйста, выскажите свое мнение.
#11. 29 августа 2011 года, 21:01. пишет:
Теория Калуцы — Клейна не согласуется в экспериментом. В ней возникают «башни» — бесконечные наборы частиц с возрастающей до бесконечности массой.

Во второй части вашего комментария содержательных идей я не нашел.
#12. 22 сентября 2011 года, 08:55. пишет:
Считается, что лептоны (в частности, электроны) не участвуют в сильных (цветовых взаимодействиях). На чем основан этот вывод? Только на отсутствии экспериментальных данных, подтверждающих это взаимодействие. Или имеются еще теоретические ограничения?
Но, например, Абдус Салам, считает, что лептонное число — это и есть четвертый цвет и, следовательно, лептоны имеют черно-белые цветовые заряды и могут взаимодействовать по аналогии с кварками. Пример — константа связи в графене, имеющая ту же величину, что и константа сильного взаимодействия. Каково Ваше мнение?
#13. 23 сентября 2011 года, 15:03. пишет:
Вывод основан на экспериментальных данных. Для описания этих данных построили стандартную модель, в которой у лептонов нет сильных взаимодействий. Стоит отметить, что до сих пор не было подтвержденных расхождений в экспериментальных данных и в предсказаниях стандартной модели.

Уверен, что Салам мог такое утверждать только лишь как аналогию. Да и то лептонный заряд — это скорее «аромат», а не «цвет»:
http://ru.wikipedia.org/wiki/Аромат_(физика)
#14. 12 октября 2011 года, 19:25. пишет:
Наблюдаемая материя трехмерна. Теория Суперсимметрии имеет дело с Суперпространством, в котором трехмерие дополняется принципиально ненаблюдаемыми измерениями. Тогда и суперсимметричные частицы не должны наблюдаться в принципе. Не так ли?
#15. 12 октября 2011 года, 22:05. пишет:
Не так. Суперпространство и суперполя можно рассматривать как математический аппарат, облегчающий построение суперсимметричных теорий. Их можно формулировать и в обычном четырехмерном пространстве-времени.
#16. 21 ноября 2011 года, 13:22. Андрей пишет:
Вы пишете:
«В квантовополевых теориях частицы материи являются „квантами“ (возмущениями) соответствующих полей. Взаимодействие между частицами переносится специальными полями. Предполагается, что частицы материи в процессе взаимодействия испускают и поглощают другие частицы — кванты поля-переносчика».
Это значит существует полевое поле на основе котороо порождаются частицы в форме возмущеий («комочков», «уплотнений», или квантовых флуктуаций) в форме фермионов с полуцелым спином, а также существует второе полевое поле и уже порожденное другими частицами — квантами поля переносчика т.е. бозонов с полуцелым спином.
Интересно, это что, матрешка? Поле частиц и поле частиц переносчиков и оба полевые поля, одно в другом, второе в первом?
#17. 21 ноября 2011 года, 13:39. Андрей пишет:
Поле частиц первично по отношению к частицам, а поле частиц-переносчиков вторично по отношению к частицам-переносчикам? Это второе поле генерируют или обуславливают частицы-переносчики т.е., верояно, бозоны?
Или два упомянутых поля изначально существуют безотносительно к актуально существующим частицам, или они частицами создаются?
А если изначальн существуют, то чем они обусловлены?
#18. 21 ноября 2011 года, 13:55. пишет:
Андрей! У вас какие-то сумбурные представления.

Во-первых, нет никакого «полевого поля». Во-вторых, у бозонов целый спин, а у фермионов полуцелый.

Чтобы представить, что такое частица как квант поля, лучше начать с электромагнитного поля. В пространстве вокруг распространяется множество электромагнитных волн: свет, сигналы радиостанций, инфракрасное излучение от нагретых тел и т. д. Всё это возмущения, «распространяющиеся колебания» электромагнитного поля (комбинации двух полей напряженности электрического и магнитного поля).

С квантовой точки зрения амплитуда волн не может быть произвольной, она меняется только на конечную (хотя и очень малую) величину. В этом смысле можно говорить, что каждая волна состоит из квантов, частиц (фотонов в случае электромагнитного поля).

В квантовой теории у электронов есть свое поле (как и у фотонов — электромагнитное). Электронное и фотонное поле не независимы, они взаимодействуют друг с другом, так что возмущения одного поля могут порождать возмущения в другом. Поэтому тут нельзя говорить о первичности чего бы то ни было.

Каждой фундаментальной частице (а фотон и электрон являются таковыми по современным представлениям) соответствует свое поле.
#19. 16 июля 2012 года, 12:08. Виктор пишет:
Значение открытия новой частицы — претендента на роль бозона Хиггса освещается специалистами неоднозначно. Хотелось бы услышать Ваше мнение.
#20. 16 июля 2012 года, 12:37. пишет:
Здесь всё просто. Есть стандартная модель, которая прекрасно описывает результаты экспериментов. В стандартной модели есть неоткрытая частица — бозон Хиггса. И вот на ускорителе находят частицу, распадающуюся как бозон Хиггса. Скорее всего это он и есть, обратное маловероятно.
#21. 22 июля 2012 года, 13:47. пишет:
Меня уже много лет мучает вопрос, что это за физика такая сверхвысоких энергий? Почему считается допустимой своеобразная экстраполяция свойств материи от «обычных» энергий в пределах 200 МэВ до десятков ТэВ и более? Почему никто не задается вопросом о том, что при сверхвысоких энергиях составные элементы «обычной» материи проявляют специфичные свойства, принципиально отличные от «нормального» состояния? Может быть, наблюдаемые свойства материи при сверхвысоких энергиях — это ее совершенно особое состояние, в определенном смысле не имеющее ничего общего собственно с материей как таковой? Может быть, в этой области энергий начинают работать специальные защитные механизмы Природы, не имеющие статистической значимости для процессов взаимодействия во Вселенной? Т.е. сверхвысокие энергии не дают ровным счетом никакого понимания структуры материи и исполнительных механизмов ее взаимодействия для нормальных условий во Вселенной: от межгалактического вакуума до процессов в центрах звезд. Такого быть не может?
#22. 22 июля 2012 года, 15:46. пишет:
Если отвлечься от деталей, то такая экстраполяция в область неизвестного вполне естественна. Однако результаты экстраполяции никто за истину не выдает. Сравнивая эти результаты с экспериментом, ученые либо расширяют область применимости существующей теории, либо предлагают новую теорию.

Если говорить о конкретных энергиях, то при переходе от сотен МэВ до десятков ТэВ происходит много всего интересного. Проявляется кварковая природа протонов, начинают играть определенную роль тяжелые частицы вроде упомянутых W- и Z-бозонов, топ-кварка и недавно открытого бозона Хиггса. Так что о простой экстраполяции говорить не приходится.

Изучение сверхвысоких энергий действительно не прольет дополнительный свет на природу большинства процессов во Вселенной. Например, процессы в центрах звезд — реакции термоядерного синтеза — протекают при энергиях (температурах) в десятки МэВ. Они хорошо известны. Но сверхвысокие энергии существовали во Вселенной на самых ранних этапах ее развития. Есть достаточное количество свидетельств (реликтовое излучение, первичный нуклеосинтез и др.) в пользу того, что Вселенная на первых этапах эволюции была очень плотной и горячей. Так что можно считать, что физика высоких энергий — это физика ранней Вселенной.
#23. 23 июля 2012 года, 02:14. пишет:
Вы прямо-таки сразили меня наповал: «…процессы в центрах звезд — реакции термоядерного синтеза — протекают при энергиях (температурах) в десятки МэВ. Они хорошо известны». Этого я не знал. Следует ли Вас понимать так, что все процессы преобразования атомных ядер от водорода до элементов группы железа и далее до трансурановых элементов могут быть рассчитаны со сколь угодно большой наперед заданной точностью? У меня почему-то до сих пор живо представление, что если это и считается, то только с помощью теорий, прочно базирующихся на вероятностных методах оценки. Т.е. существующие модели построены на основе обработки и анализа статистических данных и, по сути, являются интерполяционными. Такие модели могут иметь хорошее согласование с наблюдаемыми данными, могут предсказывать состояние систем и даже приводить к открытию новых свойств и объектов, но они все равно остаются лишь моделями описания внешних проявлений систем. Они ничего не говорят о внутреннем устройстве материи и исполнительных механизмах этих систем. На основании этих моделей можно лишь строить более или менее удачные интерпретации, как, например, модель атома Бора, которая сегодня уже, правда, считается наивной. Это не так? В этой области уже все известно?
Тогда не сочтите за идиотское упрямство, объясните, пожалуйста, почему во всех популярных изданиях практически всегда присутствуют слова: «это поможет ученым лучше понять устройство материи и нашей Вселенной». О каком устройстве материи идет речь? О той материи, которая окружает нас и из которой состоим мы сами, или о той, которая была когда-то во времена гипотетического начала всех начал?
Тут у меня возникает целый ряд вопросов пополам с эмоциями.
Эмоции в значительной степени относятся к непостижимой страсти современных ученых к теории Большого Взрыва, о которой в большинстве случаев уже и не говорят даже как о теории. Это точно установленный факт? На каком, простите, основании? Насколько мне опять-таки известно, математическая модель Большого Взрыва строилась подгонкой математических выражений и методов к фактическим наблюдательным данным и никоим образом не следует ни из какой более общей теории и тем более непосредственного наблюдения. Модель эта за время своего существования претерпела множество кризисов, которые удалось преодолеть, на мой взгляд, не совсем естественным образом. Теория Большого Взрыва была остро необходима не из общефизических, если так можно выразиться, соображений, а для того, чтобы теоретикам можно было к чему-то приложить методы анализа. Иначе имела место полная неопределенность в космологии. Но почему никто не пытается найти альтернативного объяснения реликтовому излучению и ускоренному расширению видимой материи? Что если такое объяснение существует и оно более пригодно для построения обобщенной теории?
Однако об устройстве материи. Что же вы (ученые-физики экспериментаторы и теоретики) пытаетесь найти и изучить, если «дополнительных сведений на процессы во Вселенной» пролито не будет? А нам надо знать, что было в первые секунды сотворения материи? Очень не хочется проявлять несдержанность, но вертится на языке вопрос о прошлогоднем снеге. Мне казалось, что больший интерес представляют, например, причины взрыва сверхновых или регулярных вспышек новых. До сих пор мне так и непонятно, почему между Солнцем и Юпитером (водородными шарами) присутствуют четыре небольших каменных планеты. В чем причина такого расслоения? Наконец, я думал, что упорно ищется ответ на вопрос о том, что такое гравитация. В чем отличие гравитационной массы от инертной и почему они практически равны?
#24. 23 июля 2012 года, 11:10. пишет:
Я не специалист по ядерной физике, поэтому мне придется изложить свое представление, которое может быть не вполне точным. Рассчитать ядерные реакции, как и что-либо вообще, со сколь угодно большой точностью не получится. Но в принципе для расчета нужно не так много данных. Для каждой реакции есть своя характеристика — сечение. Она показывает темп реакции. Рассчитать ее из первых принципов трудно, как и в квантовой механике трудно вычислить частоты излучения любого атома. Сечения измеряют на практике, и потом с их помощью можно подсчитать ход любых процессов, в том числе в звездах или в первые минуты жизни Вселенной.

В некотором смысле вы правы, что используется статистический метод анализа. Но он используется для определения небольшого количества параметров, а это меняет дело.

Разумеется, эксперименты на ускорителях помогают лучше «понять» устройство материи и Вселенной. Высокие энергии, недоступные современным ускорителям, присутствовали не только в ранней Вселенной. В результате разных процессов и сейчас в космических лучах есть частицы с такими энергиями. Они тоже как-то взаимодействуют. Чем эти частицы и эти взаимодействия хуже остальных частиц? Как показывает история развития физики, при росте энергии (или, что то же самое по соотношению неопределенностей, при уменьшении расстояний) физические законы и теории упрощаются. Любопытно посмотреть, что окажется дальше.

По поводу теории большого взрыва. На самом деле нет никакой теории. Большой взрыв — это просто представление о том, что Вселенная на ранних этапах была сверхплотной и горячей. С такой точки зрения это точно установленный факт, о чем свидетельствует реликтовое излучение. Оно образовалось в момент рекомбинации ионов в плазме (объединения электронов и ядер в атомы), когда ее температура была около 3000 К.

Представление о большом взрыве зародилось, когда Фридман обнаружил нестационарные решения уравнений Эйнштейна. Когда Эйнштейн сформулировал общую теорию относительности, господствовали представления о стационарной (неизменной во времени) Вселенной. Оказалось, что у уравнений Эйнштейна нет стационарных решений. После небольшой модификации такие решения нашлись, но они оказались неустойчивы. Это легко понять: если в какой-то области материи больше, то она собирается под действием притяжения. Так вот, Фридмановские решения описывают расширяющуюся однородную Вселенную. Современные космологические модели основаны на них. И первое свидетельство в их пользу — разбегание галактик — было открыто Хабблом.

Вы вот не хотите знать, что было в первые секунды «сотворения» материи. А я тогда могу возразить: «Зачем нужно изучать причины взрыва сверхновых, если они не помогут нам лучше строить дома?»

Общая теория относительности дает ответ на вопрос, что такое гравитация. Эта теория неплохо работает и объясняет наблюдения. Отклонений от нее не найдено. Однако из теоретических соображений гравитация должна согласовываться с квантовой физикой. А ОТО с ней не согласуется. Чтобы пролить свет на квантовую гравитацию, скорее всего нужны планковские энергии в 10^19 ГэВ. Они достигались, когда Вселенной было 10^-33 секунд. Но вас же не интересуют большие энергии и малые масштабы :)
#25. 23 июля 2012 года, 13:23. Владимир пишет:
Сарказм понятен и я сам виноват, что нарвался на него. Боюсь рубить с плеча, боюсь высказывать свои мысли полностью — не подумали бы люди чего дурного обо мне. Потому и выражаюсь я вот… как выражаюсь. Нет, мне интересно, что было в первые секунды. А еще мне интересно били ли эти первые секунды в том понимании, о котором сейчас только и говорят. Вы же сами сказали: неустойчивые стационарные решения уравнений ОТО после некоторой модификации. Это математика, но это никак не исполнительный механизм Природы! А Большой Взрыв — следствие интерпретации математики. Математика, безусловно, мощный инструмент. Никаких возражений против нее у меня нет, только осторожное напоминание: у любого инструмента всегда существует область его применения. Уникальный инструмент может выполнить практически единственную операцию, но с максимально достижимой точностью. Универсальный инструмент делает много, но с большими погрешностями. И всегда есть некий предел возможностей инструментов. Если угодно, это абсолютный закон Природы — не бывает бесконечностей в Природе.

Во времена Фридмана и Хаббла и даже во времена Фрэда Хойла математика, строго говоря, была единственным инструментом объективного анализа. Но сейчас есть принципиально новый инструмент, который до сих пор используется только для численного решения тех же уравнений математики. А ведь у этого инструмента есть куда более интересные способности. Вот я и пытаюсь своими вопросами понять, чем живут и что живет в головах у современных исследователей. Пока я вижу только то, что в головах практически у всех специалистов сидит только математика и что этих специалистов ни в малой степени не беспокоит прогрессивное усложнение современных теорий. Когда я учился много лет назад, я пытался разобраться в уравнениях ОТО. Не буду утверждать, что это у меня получилось в полной мере (я все-таки не физтех кончал, простой инженер авиастроения), но смысл я все же понял. Так вот, смысл современной М-теории я не способен уловить даже в общих чертах. Физики говорят: не дано, успокойся, мы и сами не все понимаем. А мне неспокойно: зачем лимиться в бесконечном упрямстве на штурм запредельных высот сложности, может, есть смысл остановиться и подумать, что что-то здесь не так? Может, сама Природа говорит, что этими методами вы только голову себе сломаете? Сложность теорий не может расти бесконечно. Безусловно, должен существовать тот или иной предел. Пусть не предел вроде стены, пусть асимптота, но сложность растет непропорционально быстрее возможностей инструмента и способностей людей его освоить. А здесь вот, рядом лежит новенький инструмент, которым вы пока только копаете. Такое может быть?
#26. 24 июля 2012 года, 22:57. пишет:
Как я понимаю, под принципиально новым инструментом вы понимаете компьютеры. По сути сейчас компьютеры очень быстро выполняют те вычисления, которые в принципе способен делать и человек. Поэтому радикальных изменений они принести не могут.

Математическая природа физических теорий обусловлена самим естественно-научным методом, в основе которого лежат измерения. Если вы хотите предсказать результаты измерений, то есть конкретные числа, вам нужна математическая теория.

По большому счету от физических теорий нужно лишь предсказание результатов экспериментов. Физикам не важно, из каких предположений делаются эти предсказания. Конечно, предпочтительнее теории с красивым математическим аппаратом, простыми исходными аксиомами, но всё это не обязательно.

Я не думаю, что физики специально строят всё более и более сложные теории. Скорее всего это неизбежное усложнение. И ситуацию запутывает полное отсутствие экспериментальных данных на тех масштабах, где важна квантовая гравитация.
#27. 18 августа 2012 года, 20:19. Аркадий пишет:
Здравствуйте, Роман.
Спасибо за статью и (особенно) за комментарии. Вы чуть не единственный популяризатор физики, который отвечает на вопросы.
Моя порция вопросов:
1) В связи с открытием бозона Хиггса (БХ) на основе уже имеющихся данных можно ли сказать, что масса легких частиц образуется по тому же механизму, что и масса тяжелых? И есть ли взаимосвязь между «легкостью» лептонов и кварков и несоответствием Стандартной модели «интенсивности» распада БХ как раз на этих каналах?
2) Не затруднит ли простым языком описать проблему иерархии. Почему именно на БХ влияние полей ведет к взлету энергии, а в случае других частиц вписывается в стандартную модель?

Спасибо за Вашу работу.
#28. 21 августа 2012 года, 00:39. пишет:
Большинство ученых сходится в том, что масса фундаментальных частиц стандартной модели (кварков и лептонов) определяется взаимодействием с неким скалярным полем, имеющем в силу некоторых причин в каждой точке пространства одинаковую величину (вакуумное среднее; выпадает в «конденсат»). Если верна стандартная модель, то бозон Хиггса — это возмущение того самого скалярного поля. В суперсимметричном варианте стандартной модели целых два вакуумных средних (за одно цепляются «верхние» кварки и нейтрино, за другое — «нижние» кварки и заряженные лептоны) и пять хиггсовских бозонов, легчайший из которых похож по свойствам на хиггсовского бозона обычной стандартной модели. Пока что слишком мало данных, чтобы понять, какой вариант реализуется на самом деле.

В любом случае, механизм не зависит от конкретных значений масс частиц. Связь между массой частиц и вероятностью их появления при распаде бозона Хиггса простая: чем легче частицы, тем труднее хиггсовскому бозону распасться на них, потому что и за приобретение массы, и за распад отвечает одно и то же взаимодействие.

Проблема иерархий заключается вот в чем. Масса частиц, как и другие характеристики взаимодействий, в квантовой теории поля приобретает петлевые поправки. Например, электрон может в некий момент испустить и тут же поглотить виртуальный фотон. Конечно, это некоторое упрощение. Но слагаемое в математическом выражении, описывающем поправки, можно наглядно представить таким образом.

Оказывается, что такая петлевая поправка бесконечна. На пальцах это означает, что чем ближе мы подбираемся к электрону, тем больше он начинает испускать и поглощать всяких виртуальных частиц. Это неприятно, но с этим можно примириться, если предположить, что собственная масса «голого» электрона равна «минус бесконечность» и, складываясь с бесконечно большой массой шапки виртуальных частиц, дает конечную наблюдаемую массу.

Но стандартная модель не может описывать физику для всех расстояний (или энергий), хотя бы потому, что не описывает гравитацию. Как вычислять поправку, связанную с «новой физикой», неясно. Ясно одно: поправка, вычисленная в стандартной модели, перестает быть бесконечной, хотя и остается большой. Теперь мы должны считать, что собственная масса электрона не бесконечная, хотя и очень большая (по абсолютной величине, по знаку отрицательная). Проблема иерархий состоит в том, что эти два числа должны быть подогнаны очень хорошо, чтобы большая поправка и большая собственная масса почти сократились и дали небольшую наблюдаемую массу.

Для простоты я объяснял петлевые поправки на примере электрона, но для него не существует особой проблемы, так как поправка к массе растет медленно, как логарифм той энергии, на которой проявляется новая физика. Единственная частица стандартной модели, для которой поправка растет очень быстро (как сама энергия) — это бозон Хиггса. Причина этих различий в конечном итоге связана со спином частиц, бозон Хиггса — единственная частица стандартной модели с нулевым спином.

Когда говорят о проблеме иерархий, приводят два масштаба энергий: 100 ГэВ в стандартной модели (в этом районе находится хиггсовское вакуумное среднее, массы W- и Z-бозонов) и 10^19 ГэВ (планковская энергия, предположительно здесь начинает работать квантовая гравитация). Сами видите, с какой точностью должны быть подогнаны числа из 19 знаков, чтобы после вычитания давать массу бозона Хиггса в 125 ГэВ.

Ответил не сразу, думал, как попроще объяснить. Будут вопросы — задавайте.
#29. 22 августа 2012 года, 05:56. Антон Никонов пишет:
Доброго времени суток.
Роман Парпалак пишет:

В частности, это означает, что при приближении к электрону на расстояния, гораздо меньшие размеров атома, начинает меняться его заряд!
При приближении чего к электрону? Любой материальной частицы?
Роман Парпалак пишет:

На пальцах это означает, что чем ближе мы подбираемся к электрону, тем больше он начинает испускать и поглощать всяких виртуальных частиц.
Опять таки, какая-то детективная история:-) — «Мы подбираемся»

Поподробней, пожалуйста.

Роман, Спасибо за популяризацию сложных идей современной физики.
#30. 22 августа 2012 года, 23:20. пишет:
Скорее это образный язык, за которым стоит соотношение неопределенностей Гейзенберга: если вы хотите исследовать малые расстояния, то вам нужны частицы с большими импульсами (и энергиями).

На опыте частицы сталкиваются друг с другом. Например, чтобы сблизить два электрона, их нужно разогнать. Чем больше энергия, тем меньше предельное расстояние между ними.
#31. 24 августа 2012 года, 13:49. Антон Никонов пишет:
Добрый день,Роман. Интересно знать ваше мнение по таким вопросам:

а) Вы читали книгу Дэвида Дойча «Структура реальности»?

б) Считаете ли Вы, что открытие в 2012 году частицы, очень похожей на бозон Хиггса, — пока главное научное событие 21-го века?

в) Другой кандидат на главное событие текущего века — превышение скорости света на 0,0026 %, тоже зарегистрированное на LHC.

г) Оба детектора (СMS и ATLAS) установили, что вероятность распада хиггсона на два фотона в 2 раза выше, чем предсказано Стандартной моделью. Если это подтвердится с достаточной статистической значимостью, то как это отразится на Вашей работе?

д) При переходе от модели Резерфорда к Стандартной модели электрон так и остался неделимым. Очень интересен вопрос о том, как же он всё-таки «раздвояется» при прохождении через две щели. Какова вероятность, что в 21 веке могут быть найдены составные части электрона?
#32. 24 августа 2012 года, 20:03. Аркадий пишет:
Роман, спасибо за ответ.
Правда я все равно не понял на счет иерархии :))
С БХ в принципе все ясно — ждем дальнейших результатов и надеемся, что человечество перестанет тратить деньги на «строительство демократии в Сирии» и построит более мощный ускоритель (хотя шансов мало — время лириков в науке прошло).
Проблема иерархии напротив, непонятна… У меня вообще в осмыслении процессов внутри ядра и (главное) взаимодействий, а также их объединения в голове сумбур полный. Во-первых представить себе поле в 3д сложно — если все в движении «тормозится» о некое закрепленное в космических координатах поле Хиггса, то почему мы не «падаем» с Земли если находимся с обратной стороны по отношению к движению? Тут на помощь приходит фраза, что поле Хиггса мешает не двигаться, а именно ускоряться, т.е. возникает производная в формулах. Все равно представить сложно, но вроде отбрехался… Все-таки всемирное тяготение и поле Хиггса это одно и тоже или вообще разные вещи?
Во-вторых, поле Хиггса как скалярное поле постоянно присутствующих в космосе частиц, но заметных только во взаимодействии с пытающимися ускориться частицами, в принципе очень наглядно и понятно обывателю. Вот если б также представить и оставшихся 3 взаимодействия. Т.е. во вселенной происходит постоянный процесс рождения и мгновенной аннигиляции переносчиков взаимодействия. Это рождает остальные 3 поля (или 1, но в 3 разных ипостасях). Сторонняя частица, помещенное в эти поля, нарушает процесс аннигиляции, облепляется переносчиками, что и образует ее характеристики (заряд, участие в сильных или слабых взаимодействиях). Вот она понятная теория :))
Но это мои фантазии — где-то читал, что только БХ незримо присутствует везде во Вселенное, других же постоянных полей нет.
Попытки же понять, как работает сам механизм (без знания формул) чувствую скоро приведет в вызову неотложки:))
#33. 24 августа 2012 года, 20:14. Арадий пишет:
П.С. к предыдущему посту.
В принципе, Вы написали, наверно, наиболее просто, как только возможно это объяснить. Просто мы (не физики) пытаемся представить себе все натурально. Это объясняет вопрос «д» Антона Никонова.
Однако, эти поправки возникают на уровне формул… Попытки же представить себе все увеличивающееся количество различных мезонов, лептонов и т.д. при «приближении» камеры к шарику с буковкой «е» на боку приводит к истерии на грани тихого помешательства :)) Хотя так все понятно — чем мощнее шандарахнуть 2 частицы, тем на большее количество новых частиц все это великолепие разлетится, а увеличение мощности столкновения для физиков и есть «приближение» камеры.
#34. 31 августа 2012 года, 11:23. Антон пишет:
Роман Парпалак пишет:

Общая теория относительности дает ответ на вопрос, что такое гравитация. Эта теория неплохо работает и объясняет наблюдения. Отклонений от нее не найдено.
Роман Парпалак пишет:

Прямым указанием на существование темной материи являются зависимости скоростей звезд в спиральных галактиках от их расстояния до центра. Эту зависимость легко вычислить. Оказывается, экспериментальные данные существенно расходятся с предсказаниями теории.
Оказывается, проще ввести темную материю, чем заподозрить, что у ОТО возможны отклонения?
#35. 31 августа 2012 года, 23:15. пишет:
Антон, книгу Дойча не читал. На остальные вопросы я ответил в отдельном материале: http://written.ru/articles/science/questions_on_physics

Туда же я вынес интересные вопросы и ответы других комментариев здесь.
#36. 10 октября 2012 года, 21:32. В. Кишкинцев пишет:
У промежуточных векторных бозонов время жизни 10 в минус 27 -ой секунды. А, какое время жизни у бозонов
Хиггса, полагаю не больше? А, Вы как считаете? Так вот за время 10 в минус 27 любой тяжёлый бозон как-то должен
заявить о своём существовании. Ведь только в этом случае можно будет считать, что бозон Хиггса или
промежуточный векторный бозон действительно образовались. На фото якобы доказывающих образование тех и 
других всё напоминает взрыв от столкновения и разлёт структур, а о том что есть и свидетели и свидетельства о их
хотя и кратковременном существовании, об этом самом главном и важном моменте почему-то, никто не пишет?
Для промежуточных векторных бозонов, правда считается, что таким свидетельством явлется, в конечном итоге,
образование лептонов и нейтрино. Однако это самые обычные продукты для всех подобных реакций даже при энергиях на порядки меньших. А, что Вы думаете на эту тему.
Однако для бозонов Хиггса и таких предсказаний нет. А, они ведь должны ещё выполнять какие-то функции
элементарных масс. Образовывать во Вселенной хиггсоновские поля, непонятно из чего состоящие, ведь какие поля
можно образовать из структур с таким малым временем жизни. Если конечно не призвать на помощь ЧТО.
Может поясните и этот момент
#37. 10 октября 2012 года, 23:31. пишет:
Уже давно новые частицы открывают по их продуктам распадов. Время жизни хиггсовского бозона действительно мало, и разделить в детекторе точку рождения и распада невозможно.

У хиггсовских бозонов есть много способов (физики говорят, каналов) распада на другие частицы, например, на два фотона, на два Z-бозона, на кварк-антикварковую пару.

Нужно понимать, что те же конечные состояния в столкновениях протонов могут образовываться множеством других способов. Поэтому здесь важно иметь теоретическое предсказание на количество рождающихся частиц.

Новые частицы открывают по отличию частоты наблюдения какого-то конечного состояния, например, двух W-бозонов, от теоретически предсказанной вероятности. Измеряя суммарную энергию конечных частиц, можно понять, какова была масса промежуточного состояния.
#38. 1 декабря 2012 года, 22:23. Игорь пишет:
Есть, кстати, у одной украинской группы Океан Эльзы альбом с названием «Суперсиметрiя» (на хохляцком одна буква «м»), после записи этого альбома из группы ушли басист, клавишник и гитарист. Судя по всему они не поняли, суть Суперсимметрии)))
http://ru.wikipedia.org/wiki/%D0%A1%D1%83% … 0%BE%D0%BC)
#39. 1 декабря 2012 года, 22:26. Игорь пишет:
У меня даже есть догадка, почему был так назван этот альбом)), лидер группы — вокалист Святослав Вакарчук учился (1991—1996) на физическом факультете Львовского университета (специализация — теоретическая физика).
#40. 23 января 2013 года, 01:59. Сергей пишет:
Непонятно только, почему физики не ищут единый закон, объясняющий «расщепление» взаимодействий. Слишком сложно?

P.S. Говорят, на LHC сейчас какие-то суперсимметричные гипотезы опровергли?
#41. 23 января 2013 года, 15:25. пишет:
При суперобъединении бегущих констант взаимодействия их пытаются свести к одному значению. По моему мнению, их следует объединить единой функциональной зависимостью. Тогда такая функция , по существу, будет ренормализующей для этих констант. Как в этом случае будет выглядить суперсимметрия? Будет ли она нужна?
#42. 23 января 2013 года, 18:59. пишет:
Сергей, теория, или, точнее, модель, которая объяснит расщепление взаимодействий, должна быть как раз той моделью, которая и будет описывать физические процессы при огромной энергии в 10^16 ГэВ (энергия великого объединения).

Основные трудности связаны с отсутствием экспериментальных данных. Ведь всё, что мы наблюдаем — это осколки единого взаимодействия, если оно вообще существует.

Сейчас мы смогли добраться только до сотен ГэВ. Как раз в этом районе нарушается электрослабая симметрия и электрослабое взаимодействие распадается на электромагнитное и слабое. За это нарушение отвечает хиггсовский механизм, который можно считать уже открытым.

Про суперсимметрию на LHC правильнее говорить, что она не открыта, ее существование не подтверждено. Какие-то варианты суперсимметричных моделей действительно исключены, хотя бы, например, конкретным значением массы бозона Хиггса. Возможность реализации других вариантов до сих пор присутствует.

Открытия новой физики еще возможны, ведь LHC всё-таки проработал не очень много, и энергия столкновений была в два раза меньше проектной.

Виктор, не совсем понял про функцию, и как константы можно объединить функциональной зависимостью. Суперсимметрия изменяет бег констант просто потому, что добавляет новые частицы — суперпартнеры. Можно предложить другие модели, в которых будут свои новые частицы и взаимодействия. Тогда константы будут бежать по-другому.
#43. 23 января 2013 года, 19:24. Сергей пишет:
> Как раз в этом районе нарушается электрослабая симметрия и электрослабое взаимодействие распадается на электромагнитное и слабое.

Вот здесь я как раз и не очень понял. Впечатление такое, что физики пытаются объединить поля не физически, а математически. То есть, грубо говоря, они пытаются описать взаимодействия между частицами единой формулой с единой суперконстантой, грубо говоря superG*q1*q2/r^2, в то же время, они совершенно забывают о том, что если электромагнитное взаимодействие, скажем, является и гравитационным, то должна быть не просто формула, но и общий физический смысл.
Грубо говоря, одна частица, либо какое-то универсальное преобразование частиц, переносящих физические взаимодействия.
Т.е. я не понимаю, почему физики не ищут способы найти преобразования частиц, переносящих полевые воздействия, друг в друга, а работают с формулами, которые где-то там может быть сойдутся по причине наличия таких преобразований.

Выше в комментариях писали про какую-то гипотезу Калуцы-Клейна. Я так понимаю, она как раз является такой попыткой, пусть и неудачной (не знаю, в чём она заключается). Есть ли ещё попытки такого рода?

> Суперпространство и суперполя можно рассматривать как математический аппарат, облегчающий построение суперсимметричных теорий.

Здесь речь идёт о каких-то дополнительных измерениях, типа оси, по которой откладывается импульс, заряд и т.п.? Или что-то ещё другое?
Можно где-то на доступном языке почитать хотя бы общий смысл этого мат. аппарата?
#44. 23 января 2013 года, 21:42. Сергей пишет:
P.S. Сейчас посмотрел, я так понял, теория струн такие вещи разрабатывает, о которых я спрашивал. Но не понял, какие там измерения и как это согласуется с суперсимметрией.
#45. 24 января 2013 года, 15:20. пишет:
Сергей, как я понял, вы под преобразованием одних частиц в другие имеете в виду превращение под действием «единых сил». Дело в том, что величина этих сил крайне низка, и вероятность переходов будет практически нулевой. Единственный эффект, который можно пытаться зарегистрировать, это распад протона (http://ru.wikipedia.org/wiki/Распад_протона). До сих пор он не был зарегистрирован.

Нужно отметить, что объединение калибровочных взаимодействий в одно — это не сведение обмена разными частицами к обмену одной частицей. Наоборот, в теориях великого объединения количество частиц и переносчиков взаимодействий увеличивается. Объединение обозначает, что разные частицы материи могут переходить друг в друга (например, электроны и кварки). То же происходит и с переносчиками (например, фотон может переходить в глюон).

В этом смысле теория Калуцы — Клейна по своей природе была такой же: гравитационное поле (метрический тензор в четырехмерном пространстве-времени) было объединено с электромагнитным в один многокомпонентный математический объект — метрический тензор в пятимерном пространстве-времени. Теория также пыталась объяснить «схлопывание» дополнительного пятого измерения, которое предполагалось вполне «настоящим», физическим.

Суперпространство — это обычное пространство-время, дополненное несколькими фиктивными измерениями. Их природа чисто математическая. По ним даже не откладывается какая-либо физическая величина. Они введены в технических целях, служат только для упрощения математических выражений в суперсимметричных моделях: такие величины как действие записываются в виде интеграла по всему суперпространству.

Рассказать больше о суперпространстве для неспециалистов у меня, к сожалению, не получится. Специалисты могут заглянуть сюда: http://susy.written.ru/2011/12/06/MSSM_math

Я не знаток теории струн, но, насколько я представляю, дело обстоит так. Дополнительные измерения в теории струн не связаны с фиктивными измерениями в суперпространстве. Скорее они похожи на дополнительное измерение в теории Калуцы — Клейна. На струнном масштабе существуют равноправные измерения, некоторые из которых компактифицируются («схлопываются»), и в результате остается четырехмерное пространство-время. Компактификация — одна из актуальных проблем в теории струн.
#46. 24 января 2013 года, 16:56. Сергей пишет:
> вы под преобразованием одних частиц в другие имеете в виду превращение под действием «единых сил».

Нет, я имел в виду то, что вы написали ниже
> То же происходит и с переносчиками (например, фотон может переходить в глюон).
Т.е. если есть единое поле, значит фотон и глюон должны состоять из каких-то одних частиц или подчиняться единому закону, пусть даже эти фотоны и глюоны виртуальные, который объясняет их различие исходя из какого-то закона.
То есть мне как раз непонятно, почему
> в теориях великого объединения количество частиц и переносчиков взаимодействий увеличивается

Если есть объединение, то все переносчики взаимодействий должны быть как-то сведены к чему-то единому. Иначе получается великое разъединение на множество полей со своим переносчиком, которые хоть и объясняют соединение и разъединение четырёх основных полей, но добавляют сущностей, нарушая принцип лезвия Оккама.

Поэтому мой вопрос заключается в том, почему физики не стремятся найти общий закон для частиц-переносчиков взаимодействий, и даже наоборот, преумножают их. И вообще, возможно ли на современном уровне как-то изучать такие переносчики в этом смысле.


> Их природа чисто математическая
Обычно математическая природа в итоге оказывается физической :)
> Теория также пыталась объяснить «схлопывание» дополнительного пятого измерения
Вот это «схлопывание», честно говоря, производит очень странное впечатление. Я так понимаю, по вашим словам, от него отказались?

> Специалисты могут заглянуть сюда
Уже непонятно, но спасибо, попытаюсь хоть что-нибудь понять.
#47. 25 января 2013 года, 14:20. пишет:
Хорошо, если вас смущает появление новых частиц при попытке описания сильных и электрослабых взаимодействий единым образом, то почему вы спокойно относитесь к трем переносчикам слабого взаимодействия (два заряженных W-бозона и нейтральный Z-бозон) и целым восьми глюонам — переносчикам сильного взаимодействия? :)

Дело в том, что если предполагать фундаментальность единого взаимодействия, которое при понижении энергии каким-то образом расщепляется на наблюдаемые нами сильные, слабые и электромагнитные взаимодействия, то переносчики взаимодействий (то же и с частицами материи) должны быть компонентами некоторой многокомпонентной величины, точно так же, как и восемь глюонных полей сами по себе объединяются в восьмикомпонентную величину.

По математическим причинам в допустимых многокомпонентных величинах места гораздо больше, чем имеющихся полей (аналогично, не всякое число глюонов вместо 8 допустимо). Свободные места должны быть заняты другими полями. Соответствующие частицы должны быть очень тяжелыми, так как сейчас они ненаблюдаемы. Возможно, что массу им придает механизм, отвечающий за нарушение симметрии и расщепление одного взаимодейстия на несколько, по аналогии с механизмом Хиггса и электрослабой симметрией.

Всё это, конечно, некоторые гипотетические модели, так как экспериментальных данных сейчас нет. В их пользу говорит принцип соответствия. Можно исходить из других предположений и получать другие модели.

От теории Калуцы — Клейна отказались по другим причинам: она предсказывает большое количество частиц, которые никогда не наблюдались. Компактификация же присутствует во многих теориях с дополнительными измерениями: http://ru.wikipedia.org/wiki/Теория_струн# … 0.B8.D1.8F
Странное впечатление она производит не только на вас. Последовательно до конца она нигде не разработана.
#48. 25 января 2013 года, 21:55. Сергей пишет:
> то почему вы спокойно относитесь
:)))
ы-ы-ы-ы, они меня бесят, смерть-смерть!!!
:)))
Я про все говорил :) .

> то переносчики взаимодействий (то же и с частицами материи) должны быть компонентами некоторой многокомпонентной величины
Вот здесь непонятно. Допустим, поля состоят из переносчиков взаимодействий, по сути, тех же полей, только более низкого уровня.
Получаем, если у нас есть много частиц, то чтобы объединить 4 поля, физики выдумывают ещё чуть ли не 20-ть субполей.
Не элегантно, мягко говоря.
Т.е. чтобы это было объединение, а не разъединение, нужно эти частицы теперь объединить в одно единое поле. Найти единую частицу, из которой состоит и глюон, и бозон и фотон.
Тогда действительно получится объединение. А так получается разъединение.

Что я не так понимаю?


> Компактификация же присутствует во многих теориях с дополнительными измерениями
В википедии по вашей ссылки указывается на аналогию в виде садового шланга. Получается забавная вещь, вроде как по этой аналогии одно из измерений пространства скручено прямо внутри других измерений. «Истинное движение муравья, ползающего по поверхности шланга, двумерно, однако издалека оно нам будет казаться одномерным»:
ведь на самом деле истинное движение будет трёхмерно. То есть в википедии уже ошибка. Издалека — одномерно, а близи сразу два измерения получается.

На самом деле совершенно непонятно, каким образом искажённое измерение вдруг получает геометрические размеры в двух других измерениях. Опять же, получается, что топология этого измерения — явно не прямая. Почему тогда не предположить, что мы живём в 3-хмерном пространстве со сложной топологией?
Как-то слишком всё это не наглядно… :(
#49. 25 января 2013 года, 23:57. пишет:
А как, по-вашему, можно найти частицу, из которой состоит и фотон, и глюон, и бозон?

На эксперименте фундаментальные частицы стандартной модели (лептоны, кварки и переносчики взаимодействий) являются точечными. Они превращаются друг в друга, но не расщепляются на нечто более простое. В отличие от атомов, атомных ядер и протонов с нейтронами.

В конечном итоге дело не в элегантности, а в совпадении предсказаний моделей и экспериментальных результатов.

В википедии нет ошибки. Движение по поверхности шланга для муравья действительно двумерно. В третье измерение он не может выбраться, хотя оно и существует. Этот пример относится к «локализации», о ней написано в следующем пункте.

Механизм возможной компактификации неясен, я об этом и написал. И у нашего пространства может быть сложная топология на планковском масштабе (http://ru.wikipedia.org/wiki/Планковская_длина)
#50. 5 мая 2013 года, 03:21. пишет:
Здравствуйте Роман. Не могли бы вы чуть более подробно обьяснить как из вакуума возникают частичы и как они исчезают. Спасибо .
#51. 5 мая 2013 года, 10:39. пишет:
Набор полей (электромагнитное, электрон-позитронное и др.) в пространстве представляет собой квантовую систему. Вакуумное состояние — это состояние с наименьшей энергией, и эта энергия отлична от нуля.

При вычислении вероятностей различных процессов в квантовой теории поля получаются математические выражения, которые выглядят так, как будто в процессе рождаются, распространяются и уничтожаются некоторые частицы, которые называются виртуальными:
http://ru.wikipedia.org/wiki/Виртуальные_частицы

Если описывать вакуумное состояние, то соответствующие математические выражения будут похожи на выражения рождения и уничтожения различных пар частицы и античастицы. При этом не происходит настоящего рождения или уничтожения частиц.
#52. 12 мая 2013 года, 03:25. пишет:
Роман, спасибо за познавательную статью!
Разрешите задать вопрос не совсем по теме и, возможно, глупый. Но меня он очень давно беспокоит. Как Вселенная может быть бесконечной, если был Большой взрыв? Ведь в таком случае мы принимаем, что в самом начале она была невероятно маленькой, меньше атома, а, стало быть, и границы имела. Потом расширялась, стала большой, огромной, но не бесконечной(?) Если границы были изначально, куда же они делись потом? Короче говоря, почему ТБВ не исключает бесконечность Вселенной?
#53. 12 мая 2013 года, 12:27. пишет:
Вопрос о границах и о нулевом моменте в эволюции Вселенной не такой простой.

В современной космологической модели, основанной на общей теории относительности, Вселенная считается бесконечной в любой момент времени, даже сколь угодно близкий к Большому взрыву. Эта модель неплохо описывает космологические наблюдения. Но действительно, согласно этой модели, непосредственно в момент Большого взрыва все расстояния нулевые.

Надо понимать, что эта трудность модели не имеет отношения к реальности, так как ранние моменты времени с огромными плотностями энергии должны описываться квантовой гравитацией, которая, к сожалению, не построена. Поэтому поведение сразу после Большого взрыва, как и вопрос о конечности Вселенной, — предмет гипотез разной степени обоснованности.

По всей видимости, материя во Вселенной простирается гораздо дальше границы видимой части Вселенной (пока не ясно, до бесконечности или нет). Эта граница образована точками, свет от которых в принципе мог дойти до Земли за всё время жизни Вселенной. Сейчас радиус видимой части Вселенной составляет 46 миллиардов световых лет и постепенно увеличивается, но это увеличение остановится из-за ускоренного расширения Вселенной.
#54. 15 июня 2013 года, 22:12. пишет:
Здравствуйте, Роман! Каким образом суперсимметрия в струнной теории реализуется только пятью различными способами?Существует ли возможность для шестого ,также связанного с остальными пятью дуальными преобразованиями?.. Просто суть теории суперструн выходит за рамки моего понимания, поэтому возник данный вопрос .
И еще …может ли гипотетический сэлектрон выступить суперпартнером не только для электрона , но и позитрона?Или принципиально невозможно, чтоб суперпартнеры для частиц были таковыми и для античастиц?
#55. 16 июня 2013 года, 12:28. пишет:
Дмитрий, я не специалист по теории струн и не могу ответить на ваш вопрос.

Частицы и их суперпартнеры имеют одинаковые характеристики, за исключением спина, отличающегося на 1/2. Это значит, что у электрона и сэлектрона одинаковый (отрицательный) электрический заряд. Суперпартнер позитрона (спозитрон?) имеет положительный электрический заряд и является античастицей сэлектрона.
#56. 17 июня 2013 года, 07:19. пишет:
Благодарю за ответ.
Значит у каждой античастицы ,возможно, существует свой суперпартнер , который является в свою очередь античастицей для суперпартнеров обычных частиц (получается этому ничто не противоречит ?).Если так, то в мире античастиц может также происходить преобразование суперсимметрии , связывающей антифермионно-(анти)бозонные поля ?Т.е. антивещество может переводится в (анти)взаимодействие? Но как быть с нейтральными частицами и переносчиками взаимодействия с нулевыми зарядами?Не совсем понимаю почему «электрически нейтральные частицы» могут и не совпадать со своими античастицами.Будут ли взаимодействия переносимые истинно нейтральными частицами в мире антивещества тождественно совпадать со взаимодействиями обычной материи ?
#57. 17 июня 2013 года, 21:19. пишет:
Про античастицы всё верно.

Существуют истинно нейтральные частицы, например, фотон, которые тождественно совпадают со своей античастицей. Суперпартнер фотона, фотино, также истинно нейтральная частица.

Электрическая нейтральность не означает истинной нейтральности частицы. Например, у нейтронов и антинейтронов нулевой электрический заряд, но это разные частицы.
#58. 6 июля 2013 года, 23:25. пишет:
Добрый день,Роман!
С понижением энергии происходит «отщепление» гравитации,сильных и слабых
взаимодействий. А не может ли до гравитации происходить отщепление
пространства(-,и)времени? Они же тоже должны квантоваться. И ,интересно, хоть
приблизительно,как бы могли выглядеть все уравнения при квантованном времени?
#59. 7 июля 2013 года, 17:12. пишет:
Можно выдвигать разные гипотезы, но экспериментальных или серьезных теоретических оснований для квантования пространства-времени нет.
#60. 12 июля 2013 года, 20:40. пишет:
Есть определенные виды мезонов нарушающих СРТ-симметрию ,т.е . нарушающих инвариантность физических явлений и приводящих также к нарушению баланса между материей и антиматерией…Но спросить хочу вот о чем ,..постараюсь сформулировать более четким образом . Процессы происходящие с этими частицами доказывают , что они как бы «различают» разницу между временными рамками прошлого и будущего.Не означает ли это что некоторые физические явления приводят к необратимости законов природы и что время имеет строго определенную направленность для обратимости таких явлений?
#61. 12 июля 2013 года, 21:22. пишет:
Современные законы природы допускают, что течение времени может не иметь строгой направленности , т.е. время рассматривается как скалярная величина .Для физических процессов и явлений симметричность такой величины принимает определенное значение…Например для термодинамики время является необратимым процессом,но
тогда как энтропия физической системы может возрастать от определенного момента в направлении прошлого?!
Сорри, что так далеко отклоняюсь от вопросов по теме )
#62. 12 июля 2013 года, 22:11. пишет:
Нужно отметить, что речь идет не о нарушении CPT-симметрии, а о нарушении CP-симметрии и связанного с ним (в силу сохранения CPT) отсутствия симметрии между прошлым и будущим. Такое нарушение действительно было открыто в странных, а затем и в очарованных мезонах.

CPT-симметрия следует из самых общих принципов: локальности, лоренц-инвариантности, то есть выполнения специальной теории относительности, и унитарности, то есть сохранения вероятности, сохранения числа частиц. На опыте не было обнаружено нарушения CPT.

Понятие «необратимости» размытое, требует уточнения определения. Например, предполагается, что при рождении во Вселенной было поровну материи и антиматерии. Однако, в силу нарушения CP-симметрии, материи стало немного больше, и после аннигиляции антиматерии совсем не осталось. Это можно рассматривать как необратимость.

Необратимость в статистической физике имеет принципиально другой характер, она не имеет ничего общего с несимметричностью фундаментальных законов относительно замены прошлого на будущее. Она связана с огромным числом частиц в термодинамических системах. При этом «совершенно беспорядочных» состояний гораздо больше, чем состояний, в которых частицы расположены в каком-либо порядке. Энтропия — это мера беспорядка в системе. Поэтому система практически всегда оказывается в состоянии с наибольшей энтропией.
#63. 12 июля 2013 года, 22:55. пишет:
Благодарю !
Первый вопрос относительно прояснился , только непонятно что же привело к нарушению СР-симметрии

По второму вопросу хочется уточнить . Что же получается ,время обрело свою направленность в результате того ,что Вселенная отклонилась от статичного состояния с низким уровнем энтропии ?
#64. 14 июля 2013 года, 13:32. Дмитрий пишет:
Здравствуйте, Роман! Большое спасибо за интересную статью!

В статье Вы пишете:
В частности, это означает, что при приближении к электрону на расстояния, гораздо меньшие размеров атома, начинает меняться его заряд! Причем такое изменение, обусловленное квантовыми эффектами, подтверждено экспериментальными данными, например, небольшим изменением уровней энергии электронов в атоме водорода (лэмбовский сдвиг).
Не могли бы Вы пояснить связь между двумя интерпретациями лэмбовского сдвига — той, о который написали Вы и той, которая обычно приводится при его объяснении ( т.е. что разность в уровнях возникает из-за взаимодействия электрона с «физическом вакуумом» электромагнитного поля (когда из-за «дрожжания» в результате взаимод. с вакуумными флуктуациями электромагнитного поля и вида кулоновсокого потенциала получается отличие полной энергии на двух орбиталях — s и p ))

Заранее спасибо!
#65. 14 июля 2013 года, 22:15. пишет:
Возможность нарушения CP-симметрии заложена в теории. Конкретная величина нарушения определяется законами. На вопрос «почему законы физики именно такие» ответить никто не может.

Вселенная никогда не была статичной. Она расширялась сразу после рождения. Выделенное направление времени проявляется и в расширении Вселенной, и в переходе термодинамических систем в состояние равновесия. Это две разных «стрелы времени». Подробнее об этом писал Хокинг: http://www.ligis.ru/psylib/090417/books/ho … /txt09.htm
#66. 14 июля 2013 года, 22:37. пишет:
Я неаккуратно написал про лэмбовский сдвиг. И этот эффект, и изменение эффективного заряда частиц вызваны радиационными поправками, или, упрощая, взаимодействием с физическим вакуумом.

Идея абзаца была в том, что наблюдение лэмбовского сдвига подтвердило «реальность» поправок и увеличило доверие к квантовой теории. Я переписал абзац.
#67. 15 июля 2013 года, 08:24. пишет:
Спасибо за ссылку на статью Хокинга .Очень интересно
Да, конечно же Вселенная не была статична, неправильно сформулировал суть вопроса…

Еще один возможно «наивный»вопрос.Теперешнее ускоренное рсширение Вселенной связывают с темной энергией.Первичному ускоренному раздуванию(инфляции) способствовало инфлатонное поле (антигравитация?)Но ведь гравитационное взаимодействие должно было усиливатся в этот момент, чтоб предотвратить полный «разлет» вещества для образования кварк-глюнной плазмы?Насколько должно увеличится гравитационное взаимодействие до полного ослабевания инфлатона?
#68. 18 июля 2013 года, 21:25. пишет:
Что такое «полный разлет»? Это какая-то надуманная проблема. Об усилении гравитационного поля во время инфляции я ничего не слышал.
#69. 19 января 2015 года, 19:22. qman пишет:
Добрый день Роман. Как вы безусловно помните, согласно теореме Нётер — каждой непрерывной симметрии соответствует свой закон сохранения. Вопрос — можно ли суперсимметрию отнести к классу непрерывных и если да, закону сохранения какой физической величины это соответствует?
#70. 21 января 2015 года, 22:07. пишет:
Да, конечно, суперсимметрия — непрерывная симметрия.

Если система полей обладает такой симметрией, то по теореме Нётер есть сохраняющаяся величина. Для нее можно даже выписать формулу (вроде (7.21) в http://www.itp.uni-hannover.de/saalburg/Le … /theis.pdf). Это новая величина, которая так и называется — суперзаряд.
#71. 27 апреля 2015 года, 16:29. Тима пишет:
Здраствуйте Роман! хотелось бы узнать ваше мнениние о природе гравитаций, и будут ли экспирименты на БАКе полезны в решение этого вопроса, если да то как?
#72. 27 апреля 2015 года, 20:06. пишет:
Мои взгляды на гравитацию стандартны для современной физики.

Сможет ли большой адронный коллайдер прояснить природу гравитации, зависит от самой этой природы :) Есть модели многомерной гравитации, в которых квантовая гравитация может наблюдаться на нем. Такие модели предсказывают рождение (за которым следует распад) микроскопических черных дыр. Подробности есть в этом обзоре: http://elementy.ru/LHC/HEP/SM/beyondSM

Однако я не являюсь сторонником таких моделей. Они слишком умозрительны.
#73. 1 сентября 2015 года, 14:59. Тима пишет:
Спасибо за ответ), надеюсь в ближайщие время квантовая теория гравитаций будит потвержденна, я канешна тоже стороник ОТО)) просто со школы ещё меня итересовал один вопрос: чем обусловленно взаимадействие гравитаций, именна физичиский смысл, а не простое искажения прос.времени,. Да сам Эйнш не смог проквантовать эту чёртову гравитацию) куда уж нам смертным
#74. 15 сентября 2015 года, 12:27. Андрей пишет:
Поясните, пожалуйста, по-подробнее, каким образам нарушение суперсимметрии влияет на работу механизма Хиггса, нарушающего электрослабую симметрию.
#75. 15 сентября 2015 года, 21:56. пишет:
Суперсимметрия — это сильное ограничение на теорию. В таких теориях хиггсовское поле не может выпасть в конденсат, его средняя величина в каждой точке равна нулю. В МССМ добавляют дополнительные взаимодействия, которые разрушают суперсимметрию. В такой модификации уже можно сделать так, чтобы хиггсовское поле приобрело в каждой точке ненулевое среднее значение (энергия системы в таком состоянии будет меньше полного отсутствия полей).
#76. 21 февраля 2016 года, 12:42. пишет:
Роман снова здраствуйте!! вот пишут что гравитационые волны обнаружены экспирементально, расскажите пжл каким образом это расширяет наши знание о гравитаций и можна ли на оснавание этих данных построить квантовую теорию
#77. 22 февраля 2016 года, 21:55. пишет:
На самом деле, экспериментальное наблюдение гравитационных волн не особо расширяет наши знания о гравитации. Гравитационные волны предсказаны Эйнштейном на основе общей теории относительности 100 лет назад. Косвенно они уже наблюдались по замедлению вращения двойных звездных систем. И проблему квантования гравитации они не решат.

Интересно, что можно попытаться построить теорию гравитации «задом наперед»: начать с квантовой теории универсальных сил притяжения между частицами, подчиняющихся закону обратных квадратов, и попытаться понять, какими могут быть уравнения такой теории. Этот подход изложен в книге «Фейнмановские лекции по гравитации». В нем оказывается, что гравитация в пределе слабых полей есть обмен гравитонами. Гравитоны имеют спин 2 и движутся со скоростью света. Уравнения теории относительности — одни из простейших возможных в этом подходе. К сожалению, таким способом нельзя построить полную квантовую теорию гравитации, свободную от проблем с перенормировками.
#78. 24 февраля 2016 года, 17:26. qman пишет:
Позволю себе немного добавить к комментарию на вопрос (#76).
Проблема квантовой теории гравитации представляет собой весьма обширную тему. Прежде всего следует отметить, что экспериментальное подтверждение существования гравитационных волн, если таковое вообще возможно никак не повлияет на решение проблемы квантования гравитации в силу того, что это два совершенно различных предмета исследования.

Теоретическое предсказание гравитационных волн вытекает непосредственно из представления системы уравнений гравитационного поля ОТО в случае отсутствия какого либо источника поля, т.е. случая свободного распространения. В этом смысле уравнения представляют собой обычную системы волновых уравнений, решением которых, в свою очередь, является обычная волновая функция потенциалов поля, или т.н. метрики.

Проблема квантовой теории гравитационного поля, как уже было отмечено, представляет существенно иную тему. Собственно проблема квантования гравитации, если так можно выразиться, заключена в самой гравитации, а если точнее, в нашем представлении о ней, а если быть ещё точнее, то в нашем представлении (в ОТО) о свободном пространстве как о непрерывной среде, в которой возможны как бесконечно большие, так и бесконечно малые длины. В этом заключается одно из ключевых противоречий с квантовой теорией, которое в конечном итоге приводит к проблеме неперенормируемой расходимости решений.

Однако если выражаться простым и топорным языком, то проблема расходимости решений возникает уже в самой КТП и вытекает непосредственно из принципа неопределенности в силу того, что любое взаимодействие в КТП имеет точечный характер. В этом смысле любая попытка следовать стандартной схеме квантования при учёте принципов ОТО (здесь, вообще говоря, следовало бы прояснить, что значит стандартная схема) заведомо обречена на провал.

Существуют однако и другие, достаточно успешные попытки включения гравитационного взаимодействия в квантовую теорию. Конкретным примером можно было бы назвать теорию струн. И хотя струнная модель НЕ является квантовой теорией гравитации, она включает ОТО как один из предельных случаев теории. Взаимодействие струн, в силу своего геометрического характера, не является точечным и потому КТП на языке струн (если так можно выразиться) свободна от расходимостей, присущих стандартной модели КТП. В этом смысле теорию струн иногда называют супер-перенормируемой теорией.

С моей личной точки зрения, огромное внимание заслуживает модель т.н. петлевой квантовой гравитации. И хотя в настоящее время эта модель имеет больше проблем нежели конкретных решений её все-таки следует рассматривать как один из весьма успешных и элегантных подходов к проблеме создания КТП с учётом гравитационных взаимодействий. Элегантность этой модели заключается в самом представлении о пространстве как о дискретной среде (отсюда и название петлевая) и собственно той естественности, благодаря которой из неё вытекает наша стандартная модель.

Также представляют интерес совершенно иные подходы к проблеме, основанные на чисто математических соображениях, а именно поиска т.н. подходящих групп симметрий, т.е. групп которые бы включали все ныне известные типы полей, включая материю.
Например: https://en.wikipedia.org/wiki/An_Exception … Everything
#79. 24 февраля 2016 года, 18:56. тима пишет:
спасибо за содержательные ответы, а вот как вам версия с «сильной гравитаций» которая может существовать в рамках суперсиметричной теорий где при очень высокой энергий возможна существование частиц с разными спинами образующих так называймый супермультиплет?, толька пжл бес этих 'суперструн'))
#80. 24 февраля 2016 года, 19:14. тима пишет:
может под действием каких то полей или механизмов (на подобие мех. Питера Хиггса) в опридиленный момент этот супермультитплет частиц начел расподаться на известные нам взаимадействия, а существовашая суперпространство раскрылась на существуещие в реальнам мире измерения?
#81. 24 февраля 2016 года, 19:29. тима пишет:
ведь существует и обратный процесс при повышений энергий на экспириментальных установках раждаются частицы с большей энергий-массой, согласна Е=м*с2 и преобразованию Лоренца, а при достижений опридиленого высот энергий, энергия-масса снова сворачивается в так наз. планковскую дырочку:-)
#82. 14 апреля 2016 года, 16:41. qman пишет:
Если честно, мне затруднительно комментировать последние три вопроса.
Сомневаюсь, что я вообще понял их…

Во первых, что касается моделей сильной гравитации… Насколько мне не изменяет память, существуют несколько попыток описать сильное взаимодействие на основе закона обратных квадратов, ни одна из которых не получила особого внимания в «mainstream» физике. Если не ошибаюсь первым был Салам, который показал, что т.к. конфайнмент может быть достигнут не отвергая закона обратных квадратов. Последнее вообще говоря противоречит выводам Квантовой Хромодинамики поскольку находится в разногласии с SU(3) симметрией, т.е. базовой калибровочной группы симметрии КХД.
Чего я собственно не понял… О какой суперсимметричной теорий идет речь?

Что касается супермультиплета, который якобы распадается на «известные нам взаимодействия» на основе схемы, подобной механизму спонтанного нарушения симметрии (если я правильно перефразировал ваш вопрос) …

Да. Подобные модели существуют и заслуживают очень большого внимания в особенности для людей, работающих в области т.н. Теории Великого Объединения.
Собственно теории пока ещё никакой нет и вообще говоря существует мало надежды, что мы её построим в ближайшие годы.
Центральная идея основана на предположении о существовании единой группы симметрии, в рамках которой возможно включение всех известных нам полей, а также четкую классификацию полей материи, подобной той, что мы имеем в нашей стандартной модели(СМ) физики, но свободной от проблем, присущих СМ, например проблемы массы нейтрино.

Работы в этой области больше напоминают математические упражнения по т.н. теории групп Ли. Идея изначально проста. Существует объединение трех калибровочных групп SU(3)xSU(2)xU(1), что соответствует нашей СМ, также существует некая схема классификации материи по принципу триплетов, синглетов, и.т.д.
Задача — найти как минимум одну такую калибровочную группу, которая включала бы SU(3)xSU(2)xU(1) в качестве своей подгруппы с минимальным количеством дополнительных калибровочных полей (опять же выражаясь топорным языком) не нарушая строгую схему классификации полей материи СМ.

Однозначных ответов данной проблеме на 14.04.2016:16:20 к сожалению нет. Существуют однако много набросок решения, которые заслуживают внимания. Одна из самых известных, это супер-симметричная модель Грегори-Глэшоу, в центре которой рассматриваются свойства группы SU(5). И хотя SU(5) включает SU(3)xSU(2)xU(1), основная беда остается в том, что SU(5) имеет ровно N^2 — 1 = 24 генератора (т.н. калибровочных бозонов), только 12 + 1 из которых являются известными нам. Это 8 бозонов SU(3)КХД + 3 бозона SU(2) теории слабого взаимодействия + 1 бозон U(1)КЭД + 1 бозон Хиггса (тот, что ответственен за нарушение симметрии в SU(2)xU(1) , т.е. спонтанный переход к U(1), при котором бозоны SU(2) приобретают массу).

Эту беду можно обойти применив схему подобную той, что мы имеем в КТП для устранения проблемы нулевой массы бозонов SU(2), т.е. механизм Хиггса. Если к тому же на теорию наложить дополнительное ограничении в виде (предмета обсуждения этого блога) т.е. супер-симметрии, то мы получим единую модель, описываемую одной группой SU(5) с тремя дополнительными калибровочными бозонами, один из которых (подобно полю Хиггса) ответственен за сужение симметрии до SU(3)xSU(2)xU(1).
Примерно так.

Вообще говоря, рассуждая на тему спонтанного нарушения симметрии (а если выражаться правильно, сужению симметрии до определённой подгруппы) необходимо отметить, что весь этот суп представляет собой не более, чем определенный математический трюк, это некие финты ушами, благодаря которым и овцы остаются целыми и волки сытыми.
Схема, благодаря которой, введение массивных калибровочных бозонов не нарушает локальной симметрии т.н. базового Лагранжиана теории.
#83. 14 апреля 2016 года, 22:44. пишет:
1. Не совсем понял, откуда вы взяли, что в SU(5) три дополнительных калибровочных бозона. Из 24 степеней свободы половина уходит в калибровочные бозоны стандартной модели. Как из оставшихся 12 получить три дополнительных калибровочных бозона?

2. Из ваших слов можно сделать вывод, что добавочные калибровочные степени свободы дают хиггсовский бозон. Но одни степени свободы калибровочных полей не могут давать массу другим степеням свободы. Лагранжиан-то должен быть калибровочно-инвариантным. Помимо полей SU(5), должны быть поля, вакуумное среднее которых будет нарушать симметрию до SU(3)×SU(2)×U(1).

Еще для неспециалистов нужно пояснить, в чем проблема 24 степеней свободы. В теории с точной SU(5) присутствует 24 безмассовых частицы. Частицы с нулевой массой легко обнаруживаются экспериментально. Но на самом деле безмассовый только фотон (симметрия U(1)) и 8 глюонов (симметрия SU(3)). Если мы хотим сохранить в основе теории симметрию SU(5), должны каким-то образом придать массу этим частицам. Механизм Хиггса дает массу трем векторным бозонам, связанным с SU(2). Можно предлложить аналогичный механизм, который сделает 12 степеней свободы очень тяжелыми, недоступными для наблюдения на современных ускорителях.
#84. 15 апреля 2016 года, 13:24. qman пишет:
:) а остальные 9 свернулись в трубочку…
да, так бывает когда пишешь не на свежую голову. Пишешь одно, думаешь о другом…
Я имел ввиду три доп. группы, т.е. 12 калибровочных бозонов, которые собственно принято разделять на два под-типа X, Y по характеру распада (хотя иногда их именуют просто X бозонами), ну и собственно скалярная составляющая отвественная за переход к симметрии СМ.

Что касается проблем SU(5)… Беда скорее не в том, что SU(5) содержит безмассовые калибровочные поля. Это собственно присуще любой калибровочной модели. Эту проблему мы научились обходить путем введения механизма спонтанного сужения симметрии до определенной подгруппы. И как вы правильно заметили в теорию можно ввести подобие механизма Хиггса, который давал бы огромные массы голдстоун-бозонам.

На мой взгляд беда с SU(5) скорее в указании на гипотетическую связь между кварками и лептонами. Собственно закон сохранения барионного числа ещё никто не отменял и на моей памяти нет ни одного эксперимента, который хотя бы косвенно указывал на возможность его нарушения. Также иерархия масс фермионных полей в SU(5) выглядит не совсем убеждающе.

Однако в целом очень красивая модель. Жаль что программист вселенной выбрал что-то другое…
#85. 25 апреля 2016 года, 10:08. тимафей пишет:
под словами «сильная гравитация» я имел ввиду — супергравитацию, а тема меня инересуют исключительно с практической точки, я экономист сам) думаю просто сколька пользы принисла бы для людей,. Вот на пример сейчас часто говорят об новом способе получение инфо. в астрофизике, путем грави.волн. В свете недавних открытий сделаных на LIRGa, ViRga, (лазерных интерф-ах).
#86. 25 апреля 2016 года, 12:54. qman пишет:
Нет. Это два различных понятия в физике элементарных частиц. Под сильной гравитацией именуют способ описания сильного взаимодействия (взаимодействие кварковых систем, конфайнмент) аналогичный тому, что описывает гравитационное взаимодействие в классической теории. Как я уже отмечал, особых успехом такой подход не пользуется.

Супер-гравитация, это… вообще говоря такая штука, простыми словами которую, описать не так просто. Формально, супер-гравитация представляет собой следствие унификации всех нам известных взаимодействий на основе требования т.н. локальной супер-симметрии. т.е. требования наличия супер-симметрии в каждой отдельной точке пространства. Подобный принцип (т.е. перенос глобальной симметрии в каждую отдельную точку пространства) является основой калибровочных теорий.

Успех при построении теории на основе требования локальной супер-симметрии заключается в том, что уравнения поля автоматически удовлетворяют свойству т.н. диффеоморфизма, т.е. независимости от выбора конкретной координатной системы. Последнее часто именуют обще-ковариатным принципом, что само по себе является основой Общей Теории Относительности (ОТО).

На сегодняшний день, супер-гравитация, представляет собой 11-мерную супер-симметричную квантовую теорию, математический формализм которой, даже в профессиональной среде, мягко выражаясь… не всем по зубам.

Одна из основных проблем теории (опуская чисто теоретические) — её проверяемость. Уровень (на сегодняшний день) достижимых энергий даже на БАКе хватает лишь для проверки низко-энергетического диапазона, в котором… (да простят меня теоретики) мы имеем лишь приближенные решения приближенных уравнений. Хотя не смотря на это, существует немалая надежда, что мы движемся в правильном направлении. Сколько лет ушло пока человечество поднялось на технологический уровень, при котором чисто теоретическое умозаключение о наличие странного скалярного поля, решившему на бумаге проблему масс бозонов слабого взаимодействия, действительно нашло свое экспериментальное подтверждение?
Это-ли не триумф нашей стандартной модели?
#87. 23 января 2017 года, 16:13. тима пишет:
Кстати на счет «проверяемости» я полностью согласен что тот уровень энергий необходимый для проверки малой суперсимметрий стандартной модели врят ли когда либо будит достигнут, но несмотря на это я бы хотел поделиться с вами своей «нелепой» для дилетанта в этой сфере, мыслью. моя Идея заключается создание «область высоких энергий» не путем разгона частиц на ускорителях, а создания искусственной « сингулярности» путем сжатие до радиуса Шварцвальда определенную область пространство, имею ввиду направленный взрыв да бы «сжать» материю до ее гравитационного радиуса, по формуле R=2Gm/c. Например для этого можно использовать термоядерный взрыв, по примеру термоядерной бомбы, где атомные бомбы служат тем самыми « Сжимателями» для создание неконтролируемой термоядерной реакций,. Я же хочу используя формулу Шварцвальда рассчитать необходимую энергию «взрыва» для создания область высоких энергий т.е сингулярность. хотя бы чисто теоретический это возможно??
#88. 24 января 2017 года, 14:28. тима пишет:
))
#89. 13 февраля 2017 года, 18:07. qman пишет:
:) …можно отправить предложение включить эту задачу в качестве одной из пяти на письменный вступительный экзамен по физики на Физтехе… в раздел термодинамики например. Если там конечно ещё проводят письменные экзамены.

Теоретически и практически это является наиболее вероятным исходом когда сверх-тяжелая звезда больше не может находиться в состоянии своего термодинамического равновесия.
Хотя спонтанный коллапс массы сопровождаемый переходом в другое агрегатное состояние (например в чисто нейтронное состояние, реакция бета-распада) вовсе не обязательно должен приводить к сингулярному состоянию.

Можно рассчитать порог (подобие критической плотности необходимой для создания реакции бета-распада во всем объеме вещества)… хотя дело скорее не в том, сколько мы затратим энергии для перевода вещества в другое агрегатное состояние и сколько именно мы получим назад. Вопрос скорее в том, что с этим делать дальше?

Что именно вы хотите использовать в качестве предмета дальнейшего исследования?
Или перефразируя вопрос, как именно вы предполагаете использовать сингулярное состояние в качестве средства достижения планковских масштабов? Излучение Хокинга…?

Кстати, это новое состояние (сингулярное или нет) вообще говоря нестабильно. Необходима энергия не меньших масштабов для его удержания.
#90. 13 февраля 2017 года, 21:46. пишет:
Хорошая шутка про вступительный экзамен :)

Чтобы подступиться к этой задаче, нужно как минимум понимать механизмы передачи энергии и вещества (какая часть энергии передается внутрь и какая будет выброшена наружу) и знать уравнение состояния при экстремальных условиях (https://ru.wikipedia.org/wiki/%D0%91%D0%B5 … 0.BE.D0.B2).

Еще, кстати, искомая энергия будет зависеть от того, сколько вещества мы собрали и что это за вещество. Если у нас есть тяжелая звезда, она сама когда-нибудь сколлапсирует, без того чтобы прикладывать дополнительную энергию.
#91. 14 февраля 2017 года, 19:10. тимафей пишет:
это просто предположение так сказать полет «фантазий» ),
#92. 14 февраля 2017 года, 22:32. qman пишет:
да… более-менее полноценное решение этой задачи потребует учесть не мало факторов и будет во многом зависит от типа вещества взятого для расчетов. Однако нас никто не заставляет решать эту проблему в лоб и для очень грубой оценки нижнего энергетического предела мы вполне имеем право абстрагироваться моделью идеального газа, состоящего например…
из точечных частиц, среднее расстояние между которыми (в начальном состоянии) составляет порядка радиуса атома водорода, а в конечном состоянии порядка комптоновской длины волны нейтрона (оставаясь при этом в том же агрегатном состоянии) . Если учесть, что сжатие происходит быстро, а также тот факт, что мы имеем дело с идеальным газом (т.е. вся потраченная энергия ушла на увеличение кинетической энергии частиц, то для расчета вполне достаточно воспользоваться адиабатическим процессом, т.е. свести задачу к чисто термодинамическим расчетам… почему бы и нет.

Отношение объемов составит порядка… ~10^-24/10^-36, т.е. ~10^12
#93. 15 февраля 2017 года, 11:49. qman пишет:
немного наврал, не 10^12, а ~10^-24/10^-39, т.е. ~10^15
т.е. линейное сокращение составит ~10^5.
Если посчитать радиус шварцшильда для солнца, то он составит порядка ~10^3 м, учитывая его настоящий радиус ~10^8 м, получаем отношение линейных размеров только для солнца ~10^5.
т.е. в идеальном приближении пока все выглядит вполне сносно…
остается взять конкретное количество газа (в начальном состоянии) и посчитать работу на сжатие.
#94. 15 февраля 2017 года, 22:15. пишет:
Я не понимаю, почему вы считаете, что в конечном состоянии среднее расстояние должно быть порядка комптоновской длины волны нейтрона. Как это связано с гравитационным коллапсом и образованием черной дыры? Мне кажется, где-то должен фигурировать гравитационный радиус.
#95. 15 февраля 2017 года, 23:33. тимафей пишет:
спасибо что начели развивать эту тему,)
#96. 16 февраля 2017 года, 00:44. тимафей пишет:
я незнаком с работами Хокинга, но насколько мне известно не какая информация не может покинуть «горизонт событий» по той причине что даже свет не может преодолеть эту область а во вселенной нечто быстрее света не может распространяться согласно СТО. даже если при этом получится сингулярное состояние я канешна незнаю что с ним дальше делать)) т.к. незнаю саму природу этого явления)), однако согласно ОТО получится «червоточина», а своим простым языком я бы сказал- так как гравитация этого объекта очень сильна, значит свет в этой области «замедляется» относительно предполагаемых двух точек, и значит все линейные характеристики этого участка должны преобразованы согласно Лоренцывых преобразований,. т.е. Пространство в этой области должно «расширяться» относительно двух произвольно взятых координат этого «пространство»,. что касается «скалапсированого остатка» я не знаю куда оно девается)), возможно оно снова «сворачивается» в некий «супермультиплет» частиц и все это сопровождается появлением и исчезновением «тяжелых» суперпортнеров, ))
#97. 16 февраля 2017 года, 01:05. тимафей пишет:
может это область и есть суперпространтство?)) где существует супермулитплет частиц с разными спинами
#98. 16 февраля 2017 года, 16:30. qman пишет:
к комментарию #94. Скажем так, с одной стороны в условии задачи действительно фигурирует гравитационный радиус в качестве конечного состояния (после сжатия), однако взяв эту величину для просчета конечного объема тела мы вернемся к проблемам, которые мы обсуждали в начале, что сделает предмет решения очень сложным, если вообще решаемым. Почему…

Во первых, мы практически ничего не знаем об агрегатном состоянии, в котором находится вещество, достигшего размеров, соизмеримых со сферой шварцшильда (существуют лишь догадки). Однако мы знаем точно, что достичь этих объемов без скачкообразной смены агрегатного состояния невозможно. Если не ошибаюсь, еще Зельдович в своей классификации эволюции звезд с массами, превышающих предел Чандрасекара указывал на тот факт, что ни одна подобная звезда не может достичь гравитационного радиуса не пройдя фазу перехода к вырожденному нейтронному газу в большей части своего объема.

Поэтому для грубой оценки искомой энергии сжатия до размера сферы шварцшильда, мы не имеем права пользоваться только законами классической физики. В частности, в последнем случае, было бы ошибкой рассматривать полный процесс как чисто адиабатическое сжатие, поскольку в конечном итоге мы имеем дело с фазовым переходом, в котором высвобождается огромное количество энергии. Полное решение потребует вовлечение квантовой статистики, термодинамику вырожденного газа, учет энергии унесенной нейтрино в реакции бета-распада и.т.п…

Для грубой оценки этого всего можно избежать, ограничившись лишь фазой сжатия, при которой газ всё ещё можно рассматривать как классический, разумеется до определенного предела… Пределом, в этом смысле, можно взять среднее расстояние между частицами идеального газа, соизмеримого с комптоновской длиной волны нейтрона. Объем в этом смысле будет соизмерим с тем, что мы имеем для нейтронных звезд.
Грубо… Да. Притянуто за уши, согласен … однако необходимо выбрать что-то, с чем можно работать.
#99. 17 февраля 2017 года, 14:15. qman пишет:
к комментарию #96. :) В рамках квантовой механики, вообще говоря, не запрещено проходить сквозь стены (я кстати сам пытался пару раз это сделать, правда не в совсем трезвом состоянии, после чего однозначно пришел к выводу, что мое тело, вопреки желаемому, все-таки в большей степени подчиняется классической статистике).

Это называют туннельным эффектом, а суть его в том, что частица всегда имеет не нулевую вероятность пройти через потенциальный барьер, энергия которого превышает полную энергию самой частицы. Вообще говоря, квантовая наука полна подобных фокусов.

Так вот, собственно этот самый туннельный эффект (предположительно) и ответственен за гипотетическое испарение черных дыр. Термин, который впервые появился в работах Хокинга. Суть в том, что пара частиц, находящихся в т.н. запутанном состоянии (кстати ещё один фокус квантОв, в споре о котором, Бор и Эйнштейн потратили лет десять своей жизни в итоге так и не придя к конкретному выводу) и находящихся вблизи горизонта событий вовсе не обязательно должны свалиться в дыру. Одна из частиц запутанной пары имеет не-нулевую вероятность покинуть дыру, причем с энергией (что очень важно) превышающей начальную энергию самой пары…

На этой основе можно заключить, что дыра на самом деле может излучать. Более того, для этого излучения можно посчитать температурный спектр, который оказывается обратно пропорциональным массе дыры. Исходя из последнего свойства можно вообще говоря делать выводы о продолжительности жизни микроскопических сингулярностей.

Сие имеет и положительный характер. Если например какая нибудь швейцарская бабушка, начитавшись новостей об успехах Большего Адронного Коллайдера подаст иск в суд на ЦЕРН, дескать ученые создают черные дыры, сами того не ведая, которые могут пожрать бабушку вместе с её приусадебным участком, то у физиков всегда есть под рукой веский аргумент, однозначно доказывающий, что после рождения, дыра испариться ещё за долго до того, как она достигнет бабушки.

Кстати в сете изложенных фактов черные дыры, более нельзя называть абсолютно черным телом.
т.е. черные дыры ни такие уж и черные… :)
#100. 17 февраля 2017 года, 15:10. qman пишет:
…интересно, иногда прочитывая прошлые комментарии можно с ужасом наткнуться на фразы, написанные своей же собственной рукой. В частности в комментарии #82 я чуть-ли не обозвал бозон Хиггса калибровочным… :) не помню что за напиток пил я тогда, одним словом прошу прощения у хозяина сайта, а также его посетителей. В пору заняться самобичеванием за ересь…
#101. 17 февраля 2017 года, 20:36. тимафей пишет:
к комментарию #94. Возможно qman затрагивая связь расстояние комптоновской длины волны нейтрона с гравитационным коллапсом, имеет ввиду создание не «Классической черной дыры» в космическом масштабе, а так называемую «планковская черная дыра» т.е низшая граница размерности длины и массы, появление которой некоторые серьезные ученые предсказывали на Баке.
#102. 17 февраля 2017 года, 22:39. qman пишет:
нет…, уверяю вас, планковскую черную дыру я точно не имею в виду :)
Для расчетов такого масштаба, не обойдешься никаким ныне существующим вычислительным арсеналом. Тут уж точно необходима полная квантовая теория гравитации…

Полагаю после проведения эксперимента по сжатию до планковской черной дыры, обанкротится не только вся земная экономика, но также большая часть видимой вселенной.
#103. 17 февраля 2017 года, 23:29. тимафей пишет:
а что кстати скажите про голографическую теорию гравитаций ? сам недавно толька услышал о такой, решает ли она существующие проблемы квантования?,
#104. 22 марта 2018 года, 16:59. Игорь пишет:
«..При понижении энергии от объединенного взаимодействия „отщепляется“ сначала гравитационное взаимодействие, потом сильное, а в завершение электрослабое взаимодействие распадается на слабое и электромагнитное…» что-то в этой последовательности не так (возможно, я чего-то не верно понял) — ведь электрослабое взаимодействие распадается по хиггсовскому механизму, в результате чего элементарные частицы обретают массу. Но массивные объекты как раз взаимодействуют с помощью гравитации. Получается, гравитация существовала до того как возникли объекты, способные гравитационно взаимодействовать? Тогда что такое гравитация?
#105. 23 марта 2018 года, 00:44. пишет:
Фундаментальные частицы стандартной модели, кварки и лептоны, действительно получают массу с помощью механизма Хиггса. Но этот механизм — не единственный способ дать частице массу. Так, масса протона объясняется его сложным устройством. В квантовой хромодинамике есть расчеты, показывающие, что при отсутствии механизма Хиггса масса протона составляла бы около 70% от настоящей.

С другой стороны, в гравитационном взаимодействии участвует не конкретно масса, а тензор энергии-импульса (упрощая, можно сказать, что это энергия и ее потоки в пространстве). Например, гравитация действует на фотоны. Луч света отклоняется в гравитационном поле звезд, несмотря на нулевую массу фотона.

Так что противоречия тут нет. Квантовые поля и гравитация существовали всегда. У полей есть ненулевая характеристика — тензор энергии-импульса. Она определяет интенсивность взаимодействия с гравитационным полем.
#106. 23 марта 2018 года, 20:19. qman пишет:
полностью согласен с комментарием #105.

на вопрос #103 (…лучше поздно чем никогда) отвечу, что подобная модель находится только в стадии разработки. Идея, если не ошибаюсь, принадлежит Хуану Малдасена, в соавторстве с Леонардом Саскиндом ему удалось показать, что (очень грубо выражаясь) струнная модель на компкактной 3-Мембране может при определенных допущениях дать перенормируемую квантовую теорию в 4-пространстве, покрытым этой мембраной, т.е. дать обще-ковариантную модель КТП.

Для справки, наша стандартная модель в настоящее время работает только в касательном 4-пространстве т.е. псевдо-евклидовой метрики (1 -1 -1 -1). Обще-ковариантная модель КТП должна быть перенормируемой на произвольной метрике. Это собственно предположительно и есть квантовая теория гравитации, как мы ее понимали изначально.

Идея голографического принципа также имеет начало (и поводимому имеет более фундаментальный характер) в работах Р.Пенроуза по т.н. теории Твисторов. Сама модель построена на принципах проективной геометрии (грубо), что само по себе решает не мало проблем.
Недавно была предложена т.н. голографическая теория твисторов, которую с некоторым допущением можно рассматривать как самостоятельную физическую модель.

Голографический принцип также имеет отношение к проблеме сохранения информации в черных дырах.
#107. 28 марта 2018 года, 13:05. Тима пишет:
спасибо за ответ ) лудше поздна чем не когда)),. Так че там о создания искусственной сингулярности в « земных условиях» каковы ваши расчеты на счет исходного материала, какова необходимая минимальная энергия сжатия до радиуса Шварцвальда и каковы теоритические последствия создания этого обьекта в земных условиях.
#108. 28 марта 2018 года, 13:14. Тима пишет:
куда тут до сингулярности) люди еще до сих пор управляемую устойчивую термоядерную реакцию в реакторе создать не могут))
#109. 28 марта 2018 года, 14:44. qman пишет:
у нас на физтехе, лет двадцать назад, не знаю как сейчас, пользовался большим успехом один задачник по физике, автор С.М.Козел с кем-то еше…
у нас тогда не редко мозжно было услышать такую фразу: «что-то подобное у Козла было…». и помоему что-то похожее действительно было в разделе термодинамики.
Проблема однако, (даже с грубым) бодсчетом упирается в многократные фазовые переходы, которым подвергается вешество на всем этапе сжатия.
#110. 28 марта 2018 года, 15:07. qman пишет:
#108 — Отнюдь, микро-сингулярности создаются периодически (на LHC например). Живут однако не долго. Управляемый термоядерный синтез, это проблема несколько иного плана, имеющая отношение скорее к нашему технологическому уровню, который сам по себе зависит от многих отрослей.
Интересно, что слиянее ядер водорода, вообше говоря, достижимо и при комнатных температурах если добиться существенного сужения сечения реакции.
Например, заменой электронов мюонами. Выход реакции однако не становится выигрышным, посколько создание мюонов требует также энергии.
#111. 8 июня 2020 года, 13:52. qman пишет:
Добрый день. Хотел поделиться с теми, кому это может быть интересным. Недавно рылся в arxiv.org и наткнулся на вот это: https://arxiv.org/abs/1502.06438

Странным образом это имеет отношение к тому, что я (несколько опрометчиво) ляпнул на этом форуме несколько лет назад. А именно… косвенно обозвал Бозон Хиггса калибровочным, поставив его в ряд с другими калибровочными бозонами СМ. Если честно, ошибочно или нет, но меня с самого начала ознакомления с принципом спонтанного нарушения симметрии не покидало чувство какой то недоделанности в идее введения скального поля, нарушение симметрии которого приводит к массовому члену в лагранжиан слабых полей.

Факт в том…, что сама неабелевость (некоммутативность) операторов SU(2) при попытке локализовать лагранжиан по отношению к SU(2) симметрии, действительно (при определенных допущениях) может приводить к появлению дополнительного массового члена, без вовлечения дополнительного скалярного поля с вакуумным ожиданием не-равным нулю ( ! ), т.е. грубо говоря, сама неабелевость SU(2) может приводить к gauge- инвариантной комбинации бозонов SU(2), которая выглядит как само-взаимодействие gauge-бозонов. Лагранжиан при этом все ещё остается SU(2) симметричным.

Кстати, конкретным примером похожего эффекта является SU(3), неабелевость которой, приводит к дополнительным членам в лагранжиане КХД, отвечающие эффектам само-взаимодействия глюонов. В случае SU(3) это однако не приводит к дополнительным массовым членам.

Вообще говоря, (если это кому интересно) сам факт того, что мы называем требованием безмассовости калибровочных бозонов, является фактом нашей (чисто-математической) неспособности удовлетворить симметрию при калибровочных преобразованиях. Массивные члены лагранжианов (просто) выкидываются.

И именно это может быть ошибкой. Поскольку введение вспомогательного скалярного поля с ненулевым вакуумным ожиданием может быть автоматически компенсировано неабелевостью более обшей группы симметрии.

Как вам такое Господа?
#112. 8 июня 2020 года, 16:08. qman пишет:
:) Не знаю понятно я выразился или нет… Я не в коей мере не отрицаю безмассовость калибровочных бозонов U(1) и SU(3). Я лишь указываю на то, что сам факт нашей неспособности удовлетворить локальную симметрию при наличии массивных членов в лагранжиане калибровочных полей вовсе не (обязательно) рассматривать как основополагающий принцип.

Компенсации массивных членов в лагранжиане можно достичь (например) введением самовзаимодействующих калибровочных полей (с массами). Последнее однако (повидимому) достижимо только в неабелевом случае и только в конкретных группах симметрий. т.е. примерно так как мы это имеем в случае с SU(2) если мы рассматриваем лоренц и gauge-инвариантную комбинацию двух бозонов.
#113. 8 июня 2020 года, 16:25. qman пишет:
…если однако принять эту возможность в случае с SU(2) мы автоматически возвращаемся к проблеме с возникновением массы фермионов в общем случае. Поскольку рассматривать Хиггс бозон как комбинацию векторных бозонов SU(2) например когда учет слабых взаимодействий не не нужен… в общем не имеет смысла…

Короче, интересная идея, но что-то тут снова не так….
#114. 8 июня 2020 года, 23:55. пишет:
https://arxiv.org/abs/1502.06438 — малопонятная статья. Откуда автор взял формулу (10)? Почему он уравнял слагаемые, квадратичные по калибровочным полям и полям Хиггса, с коэффициентом пропорциональности p? И почему этот коэффициент дальше исчез? Почему нельзя было стартовать с нормального лагранжиана, который бы чем-то отличался от стандартной модели, и из которого получились бы новые следствия? Почему надо было влазить в середину стандартных расчетов с непонятным новым коэффициентом p?
#115. 13 марта 2021 года, 23:24. пишет:
Добрый день, у меня возникла странная мысль толька не судите строго. Допустим если взять гипотический брус с двумя концами диаметром с карандаш, неважно деревяный или металический, длиною допустим в 10 миллион км и расположить его в открытом космосе так что бы два косманафта находились по обоим его концам и держали его в руках, а теперь вопрос: если один из космонафтов немного толкнет брус в сторону другого космонафта тот соотвествено почти однавременно ощютит толчок бруса, не будит ли это протеворечить это второму постулату Эйнштейна об невозможности передать сигнал или взаимодействее быстрее скоросьи света???) простити за ошибки и за этот мысленый экспиримент, хотелось бы узнать ваше мнение.

Оставьте свой комментарий


Формулы на латехе: $$f(x) = x^2-\sqrt{x}$$ превратится в $$f(x) = x^2-\sqrt{x}$$.
Выделение текста: [i]курсивом[/i] или [b]жирным[/b].
Цитату оформляйте так: [q = имя автора]цитата[/q] или [q]еще цитата[/q].
Других команд или HTML-тегов здесь нет.